Está en la página 1de 56

11.

Directors duty to avoid conflicts:


Directors fiduciary duty: 1) 2) Duty of good faith last week Duty to avoid situations where, without the consent of the company, the directors personal interest The tiers of rules affecting directors interests in transactions with their company p494

1) 2) 3)

Equitable principle to avoid conflict irrespective of fairness of transaction from companys perspective (Furs Ltd v Tomkies ) - making the transaction voidable Provisions in company constitutions modifying equitable rules Statutory duty of disclosure imposed by the Act

Replaceable rule s194: may validate transactions that would be invalidated by the equitable principle because of Ds interests in them. Restriction upon board participation by interested directors of public companies: p496

S195(1) D must not be present while matter considered at meeting or vote Except: where the interest is exempted from disclosure under s191(2) OR s195(2) other Ds pass resolution that i) identifies D & nature & extent of interest and ii) theyre satisfied interest shouldnt disqualify him.

S195(3) ASIC approves, want of quorum & compelling reason urgency If D with material person interest also possess confidential information relevant to a matter before the board disclosure and non-participation may not be sufficient in the circumstance (PBS v Wheeler ).

Aberdeen Railway v Blaikie (1854) personal interest vs. fiduciary duty= conflict of interest

Partnership sued railway company for non-performance of contract, company sought to avoid performance on grounds that D of company was also a member of the partnership. Held: Blaikie was both chairman of company and member of partnership. His duty to company is to obtain chairs at lowest price whilst his personal interest would induce him to obtain the best price possible as far as the advice he should give them, he put his interest in conflict with his duty (irrelevant if hes sole D or one of many).

No one, having such duties to discharge, shall be allowed to enter into engagements in which he has - the inability to contract depends not on the subject matter of the agreement, but on the fiduciary character of the contracting party.

Need material interest and some element of control/influence over transaction Whether the transaction/outcome was fair is irrelevant

Imperial mercantile Credit v Coleman: p498

C a stockbroker and also D of plaintiff company, contracted debentures at 5% commission, he offered P only 1.5% and did not inform P of his deal, only said he was interested in the transaction. Art 83 Constitution: officer or D shall be vacated if he contracts with company without declaring his interest at the meeting.

Held: D made adequate disclosure of interests for purposes under art 83, therefore agreement between C and P is valid. From the articles it shows that in the companys view it is better to have Ds who may advance the interests of the company by their connection and by the part which they themselves take in large money dealing, than to have someone who has no share in the transaction to advance such transactions, it is not up to the court to judgement which one is better but left to the company to form their own contracts.

The liquidators of the Imperial Mercantile Credit Association v Coleman (on appeal) - HL

Declaration of interest = states what the interest is, not stating there is one D claiming to give validity to a contract which otherwise would be invalid must show that he has complied with the provisions of the clause, here, C did not comply, if Ps board had known that C was to receive 5% and yet P, bearing all the risks would only receive 1.5% then they would have declined it.

Therefore art 83 can offer no protection, remedy = the source from which profit is derived is Constitution therefore he is liable for the whole profit which was obtained.

Transvaal lands v New Belgium Land and Development: p500

Article: 2 directors should form a quorum and no contract or arrangement should be avoided, provided a director discloses the nature of his interest, but that no director could vote on the matter.

Young, Samuel and Harvey were directors only Harvey and Young voted - Harvey failed to declare he was a trustee shareholder, and that he had just been appointed but not yet accepted directorship in D company. Company bought shares in D company, which as tightly held would cause price to inflate.

Held: As trustee shareholders, Harvey owed a duty to both the company and to the beneficiaries. Therefore, there was a conflicting personal interest. Provisions of article were not observed as Harvey had voted. Without Harveys vote, there is no quorum, and therefore contract was voidable. Fact that one of the directors (who formed the quorum) is interested as a shareholder is enough to invalidate the transaction unless Constitution permits and trustee of shares is material interest since you would make the best bargain for it.

Gray v New Augarita Porcupine Mines: p504

G had exclusive management control of N, he issued himself shares as fully paid up at a discount and traded them at much higher prices. Companys board reconstituted, found out about outstanding claims; G declared his interest and did not vote.

Held: G liable to pay N the profits from dealing. The director must declare the nature and extent of the interest and the relation of the interest to the affairs of the company. The amount of detail must depend upon the nature of the contract, or arrangement proposed, and the context in which it arises.

The directors declaration must make his colleagues fully informed of the real state of things, it is material to their judgment that they should know not merely that he has an interest but what it is and how far it goes.

The fettering of board discretions: p555

Directors must bring to bear an independent judgment in the exercise of their powers and cannot delegate discretionary powers without the sanction of the constitution, or from binding themselves as to the future exercise of powers.

However, a board will be permitted to bind themselves to do whatever is required in the future to complete a transaction now.

Thorby v Goldberg

Permissible for a board to make an agreement to allot specified shares for the purposes of bringing capital into the company, to effectuate the transaction being negotiated. There are instances, e.g. sale of land, where the proper time for the exercise of discretion is during negotiation, and not when contract is to be performed.

Ratio: The proper time for D to decide whether their proposed action will be in the interest of the company as a whole is the time when the transaction is being entered into, and not the time when their action under it is required. If at the former time they are bona fide of the opinion that it is in the interest of the company that the transaction should be entered into and carried into effect, then they can bind themselves to do whatever under the transaction is done by the board.

An undertaking given by a director that is within the scope of their contractual power and is in accordance with their duty of good faith is not an improper fetter on their future discretion, but merely the consequence of a previous exercise of that discretion.

London and Mashonaland v New Mashonaland: p557


Company sought to restrain chairman from acting as a director of a rival company, chairman had made no undertaking that he was not to become a director of a similar company, nothing in the Constitution contained such restraint.

Held: Nothing in the Constitution required chairman to give any part of his time to the business of company, not sufficient damage had been shown so no cause had bee made out There may be potential conflicts but until there is an actual conflict, these rules do not come in. Rationale: wide distinction b/w asking D to account for profit made out of his fiduciary relationship and asking D not to join board of competing company in case he breaches fiduciary duty in future.

12. Directors duty not to make secret profit


Use of position civil obligations: S182(1) Use of position: directors, other officer and employees of a corporation must not improperly use their position to: (a) Gain an advantage for themselves or someone else; or (b) Cause detriment to the corporation Civil penalty provision s1317E S183(1) Use of information: A person who obtains information because of they are, or have been, a director or other officer or employee of a corporation must not improperly use the information to: (a) Gain an advantage to themselves or someone else; or (b) Cause detriment to the corporation

Secret profits: the appropriation of corporate property, information and opportunity: p522
A fiduciary is accountable for profit arising by reason of and in the course of his fiduciary office:

1) 2)

Where the benefit was obtained in circumstances where there existed a conflict or the significant

possibility of conflict between directors duty to company and personal interests Where the benefit was obtained or received by use or by reason of the office of director or of

opportunity or knowledge resulting from it. [Chan v Zachariah]. Disclosure of Material Facts Directors CANNOT use their position to profit, nor misappropriate company property unless all material facts have been disclosed and approved by the relevant company organ. Directors are generally treated as trustees of company funds and property in their hands or under their control; and accordingly, if these are misapplied, will have to account for it. [Furs Ltd v Tomkies].

Furs Ltd v Tomkies 1936


Facts:

D was MD of P company and manager of a portion of the business, another company express interest in purchasing this portion. D was directed by the board to conduct negotiations on behalf of the P, purchaser would only acquire business if Ds services were also secured.

This was conveyed to the board who informed him that if the sale went ahead, P could not retain him on staff, and the chairman added I would advise you to make the best deal you can. During the deal, D received a payment (price sought by P was $13000, final offer was $85000, the other company paid D $5000) in addition to salary which D concealed.

Held: the undisclosed profit belongs in equity to the company

It was irrelevant that unless the D agreed to go with the business, the sale would not have taken place it was due to his failure to disclose payment that he is under a fiduciary obligation to account for it. D was able to obtain and demand the sum because he was delegated to negotiate.

D could have kept the payment if upon disclosure, the act was approved by resolution at a GM or if all shareholders acquiesced. Even if the board did purport to give D such authority, their action would be ineffectual because it would involve a breach of their duty to the company.

Diversion of Corporate Opportunities Directors will be accountable for profits to the company, if they divert business opportunities away from the company and into their own business. Directors cannot sacrifice the interests of the company which they are bound to protect [Cook v Deeks].

Cook v Deeks 1916: p526

Toronto Construction Co. formed for construction of a railway line for CPR with D, Ds brother, H and C as directors. When contract completed, CPR commenced negotiations with 2 directors of T Co - D and H for construction of another line and C excluded from any new contract, so they formed a new company, Dominion Construction Co, which carried out new contract.

D and H used voting power at Toronto to approve sale of Torontos plant to Dominion and declare that Toronto had no interest in new contract.

Held: Whilst entrusted with the conduct of the affairs of the company, D and H deliberately designed to exclude, and used their influence to do so, the company whose interest they owed first duty to.

By acting against the interests of the 1st company, D are guilty of a distinct breach of duty in the course they took to secure contract, and cannot retain benefit of contract, but holding it on behalf of company.

Factors relevant in finding contract belonging to company: preceding history of relationships, the extent to which company has been involved in the creation of contract, and issue of oppression.

2 questions: i) whether company can claim the benefit of the contract; ii) if yes, then whether SH can ratify and approve it Here, D and H breached the duty and benefits it held for the company, this is different to a situation where D had originally acquired property which he sold to co.

Profits from Office Relevant issue: whether profit was acquired by reason and in the course of their fiduciary obligations. The liability arises from the mere fact of a profit, having in the stated circumstances being made.

It is irrelevant if D acted bona fide (honesty and good intentions), in the interest of the company, breach will not depend on fraud, or the absence of bona fide intentions. It is irrelevant that Ds acted in their capacity as Ds, and not as members of the public; and if the co would otherwise be unable to exploit opportunity [Regal v Gulliver]

Regal v Gulliver: p529 prove: Ds action done in course of management + it resulted in a profit

R owned cinema centre, formed scheme to acquire lease of 2 cinemas, make it more attractive upon sale. Scheme = obtaining grant of a lease to a newly created subsidiary company, formed by R with the beneficial interest in lease enured to the benefit of R and SH.

To obtain lease, D had to personally guarantee rent until issued capital of 5000. Proposed sale/purchase and leasehold cinemas fell through, but purchase from individual SH of their shares in the subsidiary allowed D substantial profit.

Held:

Because the shares were acquired by reason and in the course of their office of directors, they were required to account for the profits. This was despite acting bona fide in the interests of R - could have been protected through resolution.

Solicitor was requested by the board of R (requested by client) to take shares the board could give company authority to its agent to exploit the opportunity personally - not liable. Chairman was a trustee, and had not derived any profit personally; the beneficiaries of the trust could not be held accountable as they were not in a fiduciary relationship to R - not liable.

NOTE: Under s182, both the chairman and solicitor would be liable, as they are deemed to hold a duty to the company. Renouncing an Opportunity General: irrelevant if the company is unable to exploit the business opportunity, e.g. due to a lack of funds [Regal v Gulliver]. However, where the board makes a bona fide rejection of opportunity through sound business judgment, there will NOT be a breach of fiduciary obligations, if these opportunities are subsequently exploited in a private capacity [Peso Silver Mines].

Peso Silver Mines Ltd v Cropper: p538 contrast to Cooks


Peso was exploring mining claims, some of which were held by Dickson, who sought to sell. Dr Aho suggested that Dickson offer claims to Peso (whom rejected offer). Aho suggested to Cropper (on board of Peso) that they should form syndicate to acquire Dicksons claims. This was purely speculative purchase. Cropper had not disclosed interest, and was asked by Peso to transfer at cost. Held: Dicksons offer did not contain any confidential information which was not available to other purchasers, and Aho approached Cropper as a member of public. Peso made a bona fide rejection of opportunity through sound business judgment, so that any later dealings with property by anyone would not be against interests.

Relevant issues that the board had renounced interest included the timing of the offer to Cropper and that the decision was not made because of lack of funds, but through strategy. Therefore, the opportunity was open to the market, and Cropper avoided liability because the opportunity arose in his non-directorial or private capacity, after termination of fiduciary duty.

Acting On a Personal Basis Fiduciary cannot escape obligations by arguing they were acting on a personal basis, rather than in their capacity as a fiduciary. It is irrelevant that the principal was unable to exploit the opportunity, and that the fiduciary acted openly and without fraud [Phipps v Boardman]. Refer to Qld Mines v Hudson.

Phipps v Boardman

Estate of Phipps contained shares in a private company (poorly managed) so board, solicitor to trustee suggested to takeover the company. The trust could not purchase the shares itself, as it was not an authorized investment; boardman had to purchase the shares.

After taking over the company, company made substantial profit. Residual beneficiary claimed that appellants held shares as constructive trustees, and an account of profit from shares.

Held:

Boardman obtained knowledge on Lester and Harris, on behalf of trustees this was special information including confidential information obtained. Knowledge obtained from fiduciary relationship, there was a conflict of duty between personal interests and fiduciary duty - boardman held shares as a constructive trustee.

Solicitors do not have fiduciary duty like trustee

Resignation to Exploit Opportunity If a resignation is prompted or influenced by a wish to acquire an opportunity sought by the company, or where the opportunity arose due to fiduciary position AND NOT from a fresh opportunity, director will be liable to account for profits [Canadian Aero v OMalley]. Fiduciary will be accountable if the profit-making opportunity is of concern and relevance to the company. Its irrelevant that company would not have been able to exploit opportunity [Industrial Development Consultant v Cooley]. In determining whether there is a breach of fiduciary duty, look at: Canadian Aero v OMalley Position of office held Nature of corporate opportunity and its ripeness, specificness and officers relation to it Amount of knowledge possessed, circumstances in which it was obtained and whether this was special or private Factor of time in continuation of fiduciary duty where alleged breach occurs after termination of relationship with company Circumstances under which relationship was terminated.

Industrial Development Consultant v Cooley: p545


P - construction services to large industrial enterprises, D (MD of P) offered Ps services to a co. which was rejected; co. approached D, as to his availability to undertake work. D prepared documents, and later told his chairmen that his health prevented him from undertaking his MD duties. D then employed by co. for project substantially similar to one which P tendered, P sought declaration that all contracts held upon trust for them by D. Held: No need to establish causation (of loss), only need to establish that benefit arose from Ds directorship.

D was liable to account for the benefit to the company information came to him whilst he was MD and which was of concern and relevant to the co. It was his duty to pass this on to the co (dut to disclose) because of his fiduciary relationship.

Differs from Peso b/c board had not renounced its interest in the opportunity, and arguably still wanted it. It is irrelevant whether or not P would have obtained benefit but for breach of trust always treated as an overriding principle of equity.

Canadian Aero Service Ltd v OMalley: p547

O and Z who were senior officers (not directors) of Canaero, pursued project on behalf of the company. O and Z formed own company, resigning from Canaero, and tendered for contract. Canaero did not submit a proposal for the bid. O and Zs detailed proposal won the contract; Canearo sued O and Z, for improperly taking the fruits of a corporate opportunity in which Canaero had a prior and continuing interest.

Held: Peso does not apply because in Peso, companys interest cased.

There was a breach of fiduciary duty because the information in regards to the contract had been obtained by the virtue of O and Z holding office. It was irrelevant whether new co. was formed for purpose of contract or whether there were variations to new tender. Breach of fiduciary duty survived their resignations - directors knew of companys continued interest.

4 issues: determine the relationship b/w O, Z and company, if theres a duty owed to co, if theres breach, liability from the breach.

Qld Mines Ltd v Hudson: p511


QM formed by AOE to acquire mining options, when drilling was complete, no co. sought to exploit the mine, Hudson was MD of QM, sought mining exploration license, using resources and name of QM in Hudsons name. This was disclosed to AOE and Qld Mines. Hudson resigned as MD, and acted on his own at own expense significant profits, Qld Mine claimed account of Hudsons profits. Held: Where interest is disclosed and Board made a decision to renounce the interest, D can retain profit. Though Hudson acted in a manner which there was a real sensible possibility of conflict b/w his interest, and the interest of QM, QM had renounced interest and assented to Hudson going it alone, at own risk and expense for own benefit.

Hudsons venture was either outside scope of trust and outside the scope of agency created by fiduciary relationship OR QM gave fully informed consent to pursue matter no further, and leave Hudson to do as he wished.

13. Statutory disclosure obligations; related party transaction


S191 (1) A director who has a material personal interest in a matter that relates to the affairs of the company must give the other directors notice of the interest unless s191(2) states otherwise.

Notice = must give details of nature & extant and relation of interest to affairs of company A personal interest includes but is not limited to a pecuniary interest it may arise from close personal relationships (R v District Council of Victor Harbour ). Material = substantial likelihood that under all the circumstances, interests would have assumed actual significance in the deliberations (Coleman v Myers )

When will notice NOT have to be given - s191(2) and includes: The interests fall within one of the exempted categories [s191(2)(a)] Company is a proprietary company and the other directors are aware of the nature and extent of the interest, and its relation to the affairs of the company [s191(2)(b)] Director has given standing notice of the nature and extent of interest under s192. Standing notice will expire if a new director is appointed and is not given notice [s191(2)(d)]

S191(3) Notice Requirements - The notice required by 191(1) must (a) Give details of:

(i) (ii) (b)

i) The nature and extent of the duty ii) The relation of the interest to the affairs of the company; AND

Be given at a directors meeting as soon as practicable after the director after the director becomes aware of their interest in the matter.

The details must be recorded in the minutes of the meeting Directors contravention of s191 will not affect the validity of any act or transaction [s191(4)] The director must declare the nature and extent of the interest and the relation of the interest to the affairs of the company. The amount of detail must depend upon the nature of the contract, or arrangement proposed, and the context in which it arises [Gray].

D of public company should give notice where they are a D of the contracting company too (Transvaal Lands ) - but interest need not be direct (Devereaux Holdings ). The Effect of Disclosure

i)

Proprietary Companies

S194 Voting and Completion of Transactions Directors of Proprietary Co (Replaceable) If a director has a material personal interest in a matter that relates to the affairs of the company AND: (a) Discloses the extent and nature of the interest under s191 OR (b) The interest does NOT need to be disclosed under s191(2) Then:

(c) The director may vote on matters that relate to the interest AND
(d) Any transactions that relate to the interest may proceed AND

(e) The director may retain benefits under the transaction even though director has interest AND
(f) Company cannot avoid transaction because of the existence of the interest.

ii)

Public Companies

S195 Restrictions on Voting Directors of Public Companies Only A director of a public company who has a material personal interest in a matter that is being considered at a directors meeting MUST NOT:

(a) Be present while the matter is being considered at the meeting; OR


(b) Vote on the matter This will not apply if: Interest is exempted from disclosure under s191(2) [s195(1A)(b)] OR Directors pass a resolution accordingly identifying the director, nature and extent of interest and its relation to the affairs of the company, AND that the directors are satisfied that the interest should not disqualify the director from voting or being present [s195(2)] Participation with approval of other directors (s195(2)) Participation with ASIC approval declaration made under s196 (s195(3))

However, this does not apply if interest is exempted from disclosure under s191(2), or if the other directors pass a resolution accordingly. Who Should The Disclosure Be Made To? - this will depend on the provisions of the constitution. Where provided, full disclosure to the board may be sufficient [Imperial Mercantile, Transvaal]. In other circumstances, full disclosure to a disinterested board may be sufficient [Qld Holdings] Otherwise, disclosure to the general meeting who must approve the transaction by ordinary resolution [Imperial Mercantile Credit Association]

Indemnification and Insurance


Where provided for in a companys constitution, directors may be indemnified or insured against certain liabilities arising from their office. Exceptions to Indemnification Provisions do not authorize anything that would otherwise be unlawful [s199C(1)] Anything that purports to indemnify or insure is void to the extent that it contravenes these provisions [s199C(2)].

10

Indemnification for Liabilities: p566


S199A(2) Indemnity For Liability (Other than legal costs) not permitted A company must not indemnify a person against any of the following liabilities incurred as an officer or auditor of the company: i. ii. iii. A liability owed to the company or a related body corporate A liability for a pecuniary penalty order under s1317G or compensation order under 1317H A liability owed to someone other than the company or a related body corporate and did not

arise out of conduct in good faith.

Miller v Miller: ratification by SH of what would otherwise be a breach of duty, does NOT breach
s199A(2) (section embargoes an indemnity against a liability) b/c ratification can never be a blanket indemnification or exemption on a prospective basis, which is prohibited by s199A(2), but rather relates to a specific and properly disclosed infractions. Indemnification, following an effective ratification does not contravene s199A(2) since theres no liability against which indemnification can operate. Indemnification Of Legal Costs S199A(3) When Indemnity For Legal Costs Not Allowed A company cannot indemnify a person against legal costs incurred in defending an action for a liability incurred as an officer or auditor of the company if the costs are incurred:

(a) In defending or resisting proceedings in which the person is found to have a liability for which
they could not be indemnified under subsection 2

(b)

In defending or resisting criminal proceedings in which the person is found guilty

(c) In defending or resisting proceedings brought by ASIC or liquidator for a court order if the grounds for making the order are established (d) Connection with proceedings for relief to the person under this Act in which the Court denies the relief

Indemnity against costs is permitted if the officer or auditor successfully defends proceedings. Company may provide a loan or advance in respect to legal costs under s212(2). Insurance - Breach is an offence of strict liability (Sch 3 applies). S199B Insurance Premiums for Certain Liabilities A company must not pay, or agree to pay, a premium for a contract insuring a person who is or has been an officer or auditor of the company against a liability (other than legal costs) arising out of:

(a) Conduct involving a wilful breach of duty in relation to the company; OR


(b) Contravention of s182 or s183.

Related Party Transactions - apply ONLY to PUBLIC COMPANIES !


Ch 2E - rules are prescriptive and cannot be overridden by a companys constitution. Prohibits giving of financial benefits to related parties of public co unless under exceptions or disclosed to GM.

S208(1) Need For Member Approval For Financial Benefit For a public company, or entity controlled by public company, to give a financial benefit to a related party of the public company:

11

(a) The public company or entity must: 1. 2. Obtain approval of public companys members (s217-227) AND Give the benefit within 15 months of approval OR The giving of the benefit must fall within an exception set out in s 210-216

(b)

Entity Controlled By a Public Company: entity = corporation, partnership, unincorporated


body, individual or trust [s9]. S50AA Control by an Entity

(1)
(2)

An entity control a second entity if the first entity has the capacity to In determining whether the first entity has this capacity:

determine the outcome of decisions about the second entitys financial and operating policies.

(a)

The practical influence the first entity can exert, (rather than the rights it can

enforce) should be considered (b)Any practice or pattern of behaviour affecting the second entitys financial or operating policies is to be taken into account However, this DOES NOT extend to instances where: Joint ventures: where the public co and an unrelated entity jointly have the capacity to determine the outcome of decisions about another entitys policies [s50AA(3)]. Public co is under a legal obligation to influence decision of another entity for the benefit of someone other than its shareholders, for example, as a trustee [s50AA(4)].

When a Financial Benefit is given: p518


S229(1) provides general principles to determine what is a financial benefit: Broad interpretation should be adopted, even if criminal or civil penalties may be involved [s229(1)(a)] Economic and commercial substance of conduct is to prevail over its legal form [s229(1)(b)] Any consideration that is or may be given for the benefit, even if adequate, should be disregarded [s229(1)(c)] whether deal fair or not irrelevant. Examples: giving/providing finance or property to the related party; sale, purchase or lease of an asset to or from the related party; receipt or provision of service; and issue of securities or grant of option to the related party (s229(3)). Financial benefits: not defined exhaustively.

Under s229(2), giving a financial benefit specifically includes: Giving a financial benefit indirectly [s229(2)(a)] Giving a financial benefit by making an agreement, whether informal, oral or non-binding [s229(2)(b)] Giving a financial benefit that does not involve the payment of money it is sufficient if it confers a financial advantage [s229(2)(c)]

There is no threshold level of financial benefit that requires shareholder approval ANY benefit that does not fall within exemption will require approval.

Definition of a related party: p518


Related parties of a public company are defined in s228 and include:

12

Controlling entities [s228(1)] = an entity that controls a public company is a related party of the public party (if A controls B then A is a related party of B; but no the other way around). Directors, their spouses and their relatives [s228(2), (3)] siblings and grandparents not included. Entities controlled by other related parties UNLESS it is also controlled by the public company [s228(4)]

Additionally, any of the party will be a related party entity IF: It had been a related party at any time within the previous 6 months [s228(5)] Has reasonable grounds to believe it will become a related party in the future [s228(6)] Acts in concert with related party on the understanding that the related party will receive a financial benefit if the public company gives the entity a financial benefit [s228(7)] (PBS v Wheeler). The definition does NOT include a subsidiary of the public company, its directors and relatives. Refer to s46 for the definition of a subsidiary.

Exceptions to the Prohibition upon Giving Financial Benefits: p519


If a transaction is exemption or approved by SH, it does not relieve directors from duties under the law, or their fiduciary duties in connection with transaction [s230]. The exceptions to the prohibition upon giving financial benefits are listed in s210-216. The principal exceptions are: Benefits given on arms length terms, that is, where the terms are no more favourable to the related party than those that would reasonably negotiated between parties at arms length [s210]. Adler v ASIC: p520 HIHC paid $10m to PEE which under effective control of Adler. ASCI claimed this transaction contravened s208 by b/c PEE was a related party of HIHC. Doesnt matter if payment was unsecured interest-free loan or payment hold on trust. General position: trustee cant derive remuneration or personal benefit from role as a trustee. This was breached and the reasonable arms length test failed too: Adler know all the facts, so payment to PEE a financial benefit that was not arms length. Remuneration of officers s211; remuneration; payment of expenses; indemnity/insurance premium

Shareholder Approval Mechanism: p521


Approval given by shareholders under s208(1) is effective ONLY if the requirements in s217227 are complied with AND disclosure and voting requirements are satisfied. These include: Preparation of an explanatory statement setting out the related parties and nature of benefit given, and the recommendation of each director [s219(1)(a)(b)] Recommendation of each director [s219(1)(c)] Statement must contain all information known to company or directors that is reasonably required by members to decide whether it is in the companys best interest [s219(1)(e)] A vote must not be cast by the related party, or on their behalf [s224(1)]. Must lodge with ASIC 14 days before notice given to members.

13

Remedies for Breach

If a transaction breaches s208, it does not affect the validity of any contract or

transaction connected with the giving of the benefit AND the public company is not guilty of an offence [s209(1)].

Contravention of s208 = contravention of s209(2), a civil penalty provision under Pt

9.4B and s1317E will operate. Involvement in contravention is defined in s79. If the involvement in the contravention of s208 is dishonest, then an offence will have

been committed [s209(3)]. The criminal penalty that can apply is a fine of up to $200 000, 5 years or both [Sch 3].

14. Directors statutory duty to prevent insolvent trading


Part 5.7B, Div 3 Ds duty to ensure company does not trade while insolvent. NB: s436A (1) D can appoint an administrator if company likely to become insolvent at some future time. Or else D is personally liable in respect of debts incurred during the period of apprehended insolvency. Elements of the Duty: p436 S588G (1) Directors Duty to Prevent Insolvent Trading

(a) Duty applies only to a person who was D at the time when the company incurred a debt
(b) Company must have been insolvent at this time, or became insolvent by incurring that debt

(c) Reasonable grounds for suspecting that company was insolvent or would become insolvent
S588G (2) Contravention of Insolvent Trading Provision D must be aware at the time that (a) there are grounds for suspecting insolvency OR that (b) a reasonable person in a like position in a company in the companys circumstances would be so aware.

Where the failure to prevent the company from incurring the debt was dishonest, this will constitute a criminal offence, and attract criminal remedies [s588G (3)] - Refer to 5.11.2

i)

Debts Engaging Duty - Debt must be ascertainable and signifies an obligation sounding in the

payment of a sum of money or moneys worth.

ii)

Definition of Insolvency

Company is deem to be solvent = ability to pay all its debts when they fall due, looking at its cash flow position [s95A(1)]. A company which is not solvent, is said to be insolvent [s95A(2)].

14

iii) Reasonable Grounds for Suspecting Insolvency


Test - objectively reasonable grounds which must be judged by the standard appropriate to D of ordinary competence. The establishment of liability is not contingent on elements personal to the respondent. [Metropolitan Fire Systems v Miller].

iv) Directors Awareness, Actual Or Imputed Of the Companys Financial Position


If D has actual knowledge of facts or circumstances concerning the companys financial condition, this may comprise reasonable grounds for suspecting insolvency, even if the director fails to appreciate their significance or implications.

Defences to Liability for Failing to Prevent Insolvent Trading

v)

Reasonable Grounds to Expect Insolvency

S588H (2) Reasonable Grounds to Expect Company Solvency D has to prove that at the time debt was incurred, he had reasonable grounds to expect solvency and would remain solvent even if it incurred that debt.

Defence is cast in terms of expectation of insolvency this is interpreted as importing a greater degree of certainty than mere hope or possibility, and of a higher order that suspecting [3M Australia v Kernish] - The exculpation standard is higher than that of primary liability

vi) Reliance upon Information As To Solvency Provided By Another


S588H(3) Information As To Solvency From Other Person Where D establishes that at the time debt was incurred, s/he:

1.

Had reasonable grounds to believe and did believe

(i)

That a competent and reliable person was responsible for providing to D adequate

information about whether the company was solvent (ii) Other person was fulfilling that responsibility AND 2. D expected on the basis of information provided by the other person that the company was solvent at that time and would remain solvent even if it incurred that debt and any other debts that it incurred at that time.

vii) Non-Participation in Management Due To Illness or Other Good Reason


Under s588H (4), D may establish that at the time when the debt was incurred, they did not take part in the management of the company. However, non-participation must be because of illness or other good reason precluding participation. However, if D would not have been more involved in management if not disabled by illness, then it would not constitute a valid defence.

15

viii)Reasonable Steps to Prevent Incurring Of the Debt


S588H(5) defence: D establish that they took all reasonable steps to prevent the company from incurring the debt. Regards may be had but not limited to: i) any action the person took with a view to appointing an administrator of the company ii) when the action was taken AND iii) results of that action [s588H(6)] Remedies: p445 The remedy for breach of the general law duty of care and diligence, and its statutory complements is the award of damages to compensate the company for loss caused by the breach, restoring the plaintiff to a position as if the loss had never occurred Breach of s180, is a civil penalty provision refer to 5.11.1. The duty is one owed by individual Ds and officers who are personally liable for its breach. Does not set aside a board decision taken by Ds in breach of duty of care, or restrain action pursuant to such a decision this may be obtained for a breach of duty of good faith. However under s1318, the court may excuse the breach if it is shown that the defendant acted honestly. Refer to 5.9. Compensation Remedies With Respect To Insolvent Trading For an insolvent company, remedies may include: Upon contravention of s588G (2), court may order compensation on application for civil penalty order [s588J]. This applies only where the debt is wholly or partly unsecured1 [s588J (1)(b)]. Compensation proceedings founded upon contravention of s180(1) under s1317H. Under s598, court may make orders as it thinks appropriate against a person, where a corporation has suffered, loss or damage as a result of fraud, negligence, default, breach of trust, or breach of duty. These remedies provide for recovery at the suit of the liquidator and in some circumstances of individual creditors. The liability of a holding company for insolvent trading by a subsidiary company: p445 An action for compensation may be brought against a holding company where it allows a subsidiary company to trade while insolvent. 588W: subsidiarys liquidator can sue holding for compensation if:

1. 2. 3.

588V(1)(a) company must be the holding company when subsidiary incurs debt Subsidiary must be insolvent at that time or become insolvent by incurring that debt or by incurring at that time debts including that debt s588v(1)(b). Reasonable grounds for suspecting company insolvent s588V(1)(c)

There will be a contravention of the directors duty to prevent insolvent trading, but will not trigger civil recovery remedies.

16

4.

The holding company or its D(s) is aware when the debt incurred that there are reasonable grounds for suspecting insolvency (s588V(1)(d)(i)). OR Having regard to the nature and extent of the holding companys control over the subsidiarys affairs and other relevant circumstances, it is reasonable to expect either that holding company in the companys circumstances would be so aware or that its D(s) would be (s588V(1)(d)).

Liquidator can recover compensation where comp contravened Pt5.7B div 5 & causation Defences: s588X

Deputy Commissioner of Taxation v Clark [2003] NSWCA: p447


Both Mrs C and her husband are directors of SCI (pty ltd), company was wound up and liquidator obtained order against DCT for recovery of payments made by SCI by way of group tax.

Management skill of D: vary depending on the particular skills of the director Core requirement is determined by objective test S588G and s588H based on assumption that D would participate in management of Co, it follows that failure to participate for whatever reason is NOT necessarily a good reason, need to construe good in the total context.

Therefore total failure to participate and total reliance on husband are not good reasons.

15. Shareholders Remedies (1)


Note: general law actions under the fraud on the minority doctrine and standing rules have been abrogated by the introduction of the statutory derivative suit. Member of a company = member in registration or agrees to become a member of the company subsequent to registration and has their name entered on the register of the members - s 231 The protection of minority SH: p572 equitable constraint + grant of legal standing The corporate will is identified with the decision of a majority of members, by value of shareholding. However, a court may interfere with the majoritys decision, if it can be shown that the majority voted for a purpose outside an implied range of purposes for which the power to vote was conferred. Thus, there is an implied obligation not to use the power for an ulterior purpose (Gambotto v WCP LTD).

Doctrine will apply even if the procedure or action is actually legal - court will apply equitable constraint on how majority exercises power (Majority cannot act oppressively towards minorities or use power for improper purpose or else it is fraud on minority).

This is not a fiduciary duty shareholders do NOT owe fiduciary duties to each other It will tend to apply in unlisted, proprietary companies, as there are little opportunities for SH to exit the investment.

Another mean to protect minority SH granting them legal standing to litigate: Wrongs done to them personally (personal, individual or direct) no greater difficulty, cause of action = personal rather than corporate.

17

Wrongs to the company itself (derivative suit) greater difficulty, requires court approval prior commencing legal action for the company

Alteration of the companys constitution that prejudice SH rights: p585 S136 empowers a company to alter its constitution by special resolution, subject to any express requirements contained in the constitution. The power is further constrained by the doctrine of fraud upon the minority. Where amendment to the constitution gives rise to a conflict of interests, in general, an alteration of the articles by special resolution regularly passed will be valid unless: [Gambotto v WCP Ltd].

It is ultra vires, that is, beyond any purpose contemplated by the articles OR Oppressive in its usual meaning for the purposes of the Corporations Act.

Greenhalgh v Arderne Cinemas Ltd [1951]: p587


Facts: Companys constitution contained provisions prohibiting sales of shares to non-members so long as members are willing to purchase, M wished to sell his controlling interest to outsider, went under special resolution and changed the Constitution. Corporate purpose every SH was given the right to the bidder although lost the right to buy. G (minority SH) challenged the resolution as fraud on minority). Held:

SH cant expect constitution to remain the same, changes to Constitution will always cause harm to one SH or another, only when alteration discriminates b/w types of SH then it will be invalid. Decision did not discriminate b/w types of SH and the resolution not impeached, the price was fair. Special resolution those who passed it has to be in good faith, bona fide for company as a whole means that SH must proceed upon what in his honest opinion is for the benefit of the company (whole = corporate as a general body).

Australian Fixed Trusts v Clyde Industries 1959: p589

P sought to prevent amendment of the constitution, which would disenfranchise (deprived of voting rights) any member who held shares as a trustee, unless the majority of beneficiaries had directed the trustee as to the manner in which the vote was to be cast.

Held: the amendment focused on trustees in a discriminatory manner, and would create a burden on trustee to fulfil requirements; and thus no reasonable person could decide that the amendment was for the benefit of the company as a whole - The stronger the discrimination, the stronger the corporate purpose needs to be.

If the right of SH custodian (trustee) to vote has been taken away of its substance, the right would have been reduced in effectiveness. So the other SHs rights are more effective and valuable, the effect of the article is to discriminate b/w majority and minority SH.

The decision was based on prior test of whether act was bona fide in interests of company and whether it discriminated between majority and minority shareholders.

Gambotto v WCP Ltd 1951


Capital of WCP was 99.7% held by wholly owned subsidiaries of IEL. This could not be compulsorily acquired under compulsory acquisition laws due to its structure. Articles were amended to permit majority shareholders to acquire shares which they were not entitled to at $1.80.

18

This was agreed to by minority SH, except P, who did not attend meeting or vote. The shares were valued at $1.365. Appellants agreed valuation was a fair and independent one, but failed to account for tax benefits which would accrue factoring in tax losses which could be claimed. Appellants wished to retain shares. Held: Note that compulsory acquisition Pt 6A.2 Div 1

Majority: decision did not discriminate b/w SH (anyone who wanted to get out at that price could and anyone who preferred to stay in could stay in). There was no business activity at risk, nor was full ownership required for a regulatory regime. The taxation and administrative benefits on its own were not sufficient to constitute a proper purpose. McHugh differed to the content of the purpose restraint: held that the compulsory acquisition of shares is valid, only if it will enable the company to pursue some significant goal, or to protect from some action, that is external to the company. Therefore, the alteration was proper as it enabled the company to save $4M in taxes.

However, McHugh held that whilst the price offered might have been fair, there was no attempt at full disclosure, so that the majority SH did not deal with each appellant fairly. The courts decision was framed in broader terms, applying to amendments to allow expropriation by the majority of valuable proprietary rights attaching to the shares of the minority. Is expropriation valid? Only if i) Exercisable for proper purpose and ii) Exercise will not operate oppressively

Sons of Gwalia Ltd v Margaretic 2007 HC:

Facts: M purchased shares in Sons of Gwalia Ltd, few days later, company went into administration and M's shares worthless. M claimed company failed to disclose material adverse information under the continuous disclosure rules and had misled/deceived him into purchasing shares.

The administrators and company's creditor applied to FC seeking a declaration that, because he was still a SH, M's claim not provable or, alternatively, was postponed until all debts owed to creditors had been satisfied, under s563A CA.

HC: SH with a claim under a statute against a company for misleading or deceptive conduct, or for failure to comply with its continuous disclosure obligations (each a shareholder claim), could prove in the administration or liquidation of that company in respect of the damages for which the company was liable, and that this applied whether SH acquired the shares by subscription or purchase. This applied even though his loss did not crystallise before the administration.

Issue: meaning of circumstances giving rise to which occurred before the date of administration, b/c M purchased shares before company went into administration, but not sold his shares before that date and crystallised his loss, not before the administrators were appointed, but on their appointment.

Hayne J: emphasised that the statute had changed in 1992 (after the facts in Webb Distributors), broaden the claims admissible to proof on a winding-up. It was not necessary for Ms loss to have crystallised in order for him to have a provable claim, i.e. not necessary for his cause of action against the company to have accrued in order for him to have a provable claim.

19

Duties to Individual Shareholders Directors will hold fiduciary duties to individual SH where there is a relationship of confidence and trust. This will depend on the special circumstances and nature of the relationship of each individual case, but will generally arise where there is: Dependence upon information and advice Existence of a relationship of confidence Significance of the transaction for parties Extent of any positive action taken by or on behalf of directors to promote it [Coleman v

Myers ] Coleman v Myers 1977- p561

Myers family - K patriarch and chairmen, whilst his son D was MD. D made takeover offer through a company solely owned by him (funded by companys capital dividend, sufficient to discharge borrowings to finance acquisition and still yield a company with assets).

K and D advised SH to accept the offer, denying there would be any liquidation of the companys assets. They failed to disclose that the company had liquid assets available for distribution of dividends, or that real estate could be realized for sums in excess of values shown in the books.

Held: The court found that directors owed a fiduciary duty to individual shareholders due to:

Asymmetry of information (depth of knowledge and experience on one side) Family company - significant trust and confidence was reposed in the defendants, especially in K. The magnitude of the capital gains at stake was very large in the context of the value of the company D actively developed, promoted and recommended the takeover proposal Obligation to disclose material matter, material = substantial likelihood that a reasonable SH would consider it important in deciding how to vote. Price offered not fair, recommendation not made in reasonable care for interests of SH.

There was a duty of care in respect of formulation of recommendation K and D breached their obligation to disclose material matters to SH to the proposed dealing, which could affect decision.

Brunninghausen v Galvanics
G and B were brothers-in-law and directors in a small pty co. G was majority SH and an active director, whilst B was passive (minority SH and director). G did not tell B that he was negotiating on behalf of the company for sale of its business for price in excess of that reflected in share price in agreement for the sale of Bs shares to G. Held:

Fiduciary duty existed due to Bs entire dependence on G for information and advice, in regards to the negotiations. There was also an expectation that B felt he would not be cheated by nondisclosure.

D would owe a fiduciary duty to SH during negotiations for a takeover, requiring directors to loyally promote the joint interests of ALL shareholders this fiduciary duty is not a comprehensive fiduciary duty but one that was limited to the disclosure of the unexpected offer by 3rd party to purchase entire business.

The limited action by D necessary to negate the effect, in particular circumstances of his taking advantage in an unconscionable way of superior position occupied by him as against the plaintiff.

20

16. Shareholders Remedies (2)


A derivative action may be brought by member/SH(s) on behalf of a company and in the companys name based on a cause of action vested in the company [s236(2)]. Courts permission required to bring a derivative action b/c: Foss v Harbottle (abolished by s236(3))

The company should be able to decide whether to bring proceedings (proper plaintiff rule) AND The court should not intervene with the internal affairs of a company (internal management rule)

There are 4 exceptions to Foss v Harbottle rule:

1.

Fraud upon minority: abolished by s236(3)

Control requirement: where person against whom relief is sought control the majority and will not permit an action in companys name; and Fraud: majority/D benefited personally

2. Special Majorities: Where corporate acts carried out ordinary resolution but for which a special majority is prescribed by statute or constitution (Baillie v Oriental Telephone and Electrical Co) 3. Personal rights:

Personal rights conferred by statute or constitution: right to vote (Pender v Lushington), to restrain those not valid Ds as acting as such (Link Agriculture v Shanahan)

4. Illegal and ultra vires acts: Corporate acts which are illegal (Drown v Gaumont-British Picture)

21

The Statutory Derivation Action: p603 - Pt 2F.1A


Who Can Bring an Action? S236(1) A person may bring proceedings on behalf of a company if: not ASIC, auditors/creditors (a) The person is:

(i) A member, former member, or person entitled to be registered as a member, of the


company, or related body corporate OR Give the benefit within 15 months of approval OR (ii) An officer, or former officer of the company, AND

(b)

The person is acting with leave granted under s237.

Proceedings brought on behalf of a company must be in the companys name s236(2)

Pt 2F.1A applies beyond derivative actions, such as a breach of Ds duties. It allows a person to obtain leave of the court to: Bring proceedings on behalf of the company Intervene in any proceedings to which the company is a party for the purpose of taking responsibility on behalf of the company for those proceedings, or for a particular step in those proceedings [s236(1)] Therefore, it applies to actions brought by the company against 3rd party, or to defend actions brought by 3rd party against the company. Criteria for Granting Leave Strike a balance between the need to provide an avenue for redress on behalf of a company which it is unable to obtain itself and the need to prevent actions that are vexatious or without merits. S237(2) The Court must grant the application if it is satisfied that: (a) It is probable that the company will not itself bring the proceedings, or properly take responsibility for them, or for the steps in them; AND

(b) (d)
AND

The applicant is acting in good faith AND

(c) It is in the best interests of the company that the applicant be granted leave AND
If the applicant is bring leave to begin proceedings, there is a serious question to be tried

(e) Either:

(i) At least 14 days before making the application, the applicant gave written notice to the
company of the intention to apply for leave and of the reasons for applying OR (ii) It is appropriate to grant leave even though subparagraph (i) is not satisfied

Rebuttable Presumption against Unrelated 3 rd parties (anyone not D or company) There is a rebuttable presumption that granting leave is not in the best interests of the party where: Company has decided not to bring or defend proceedings AND Directors who participated in decision satisfy conditions of business judgment rule [s237(3)] S237(3) operates in aid of s237(2)(c) by creating a rebuttable presumption in specified circs, where D have made a decision of a kind that could not be made by them if the company were in liquidation [i.e. rebuttable presumption cannot arise under s 237(3) if company in liquidation but that does not mean court cannot exercise its discretion under s237(2)].

22

Independent Person S241 gives the court the power to appoint an independent person to investigate and report to the court on the Financial affairs of the company Facts giving rise to the cause of action OR Costs incurred in the proceedings [s241(1)]

The person is entitled to, on reasonable notice, to inspect any books of the company for any purpose connected with their appointment [s241(2)]. It enables the court to determine independently of the parties whether the complaint discloses a good cause of action. Permission to Discontinue or Settle Proceedings Under s240, proceedings brought or intervened in with leave must not be discontinued, compromised or settled without the leave of the court.

S239(1) If the members of a company ratify or approve conduct, the ratification or approval:

(a) Does not prevent a person from bringing or intervening in proceedings with leave under s237 or
applying for leave under that section AND (b) Does not have the effect that proceedings must be determined in favour of the defendant, or that an application for leave must be refused.

S239(2) Court To Decide What Order Or Judgment If members of a company ratify or approve conduct, Court may take ratification or approval into account in deciding what order or judgment to make in proceedings, having regard to: (a) AND How well-informed about the conduct the members were in deciding whether to ratify

(b)

Whether the members who ratified the conduct were acting for a proper purpose

S242 Power of the Court to Make Costs Orders The Court may at any time make any orders it considers appropriate about the costs of the following persons in relation to proceedings brought or intervened in with leave under s237 or an application for leave under that section: (a) (b) (c) Person who applied for or was granted leave; The company; Any other party to the proceedings or application.

An order under this section may require indemnification for costs.

Court may make orders as appropriate, according to the merits of the case a frivolous case may be subject to a costs order, or the court may apportion costs as it sees fit. Where a SH successfully prosecutes a derivative suit, the benefits of the suit are enured to the corporation. Thus, other than the courts ability to apportion costs, SH actions are inhibited due to the disparity between the personal burden of prosecution, and corporate character of resulting benefits.

23

Since corporate regulation assumes that SH will be the primary monitors of corporate management, and a significant enforcement agency, the disincentive inherent in bringing an individual suit for collective gain is a fundamental problem. Statutory Right to Injunction to Prevent Breach of CA S1324(1) Power To Make Injunction Where a person has engaged, is engaging or is proposing to engage in conduct that constituted, constitutes or would constitute: (a) A contravention of this Act, OR (b) Attempting to contravene this Act, OR The court, may on the application of ASIC, or of a person whose interests have been, are or would be affected by the conduct, grant an injunction on such terms as the Court thinks appropriate, restraining person from engaging in the conduct, and if desirable, requiring that person to do any act or thing. S1324(10) Payment of Damages Where the Court has power under this section to grant an injunction, the Court may, either in addition to OR in substitution for grant of an injunction, order that person to pay damages to any other person. Standing on Minority Shareholders

The combined operations of Pt 2D.1 and s1324 may confer standing upon minority SH to restrain directors from breaching their duty of care, or fiduciary duties. Pt 2D.1 imports fiduciary obligations of general law thus, if a minority SH is a person whose interests are affected under s1324(1), contravention may possibly found an action under s1324(10).

However, power under s1324 is discretion so an applicant only has a right to apply and not a right to an injunction [Re Brunswick].

Swansson v Pratt 2002: p605 Facts: S (SH/D of RAPP, ex-wife of H) sought leave under s237 to bring proceedings to have H (former D) to compensate RRAP for funds benefited H personally. P (D of RAPP) opposed proceedings b/c disputed payment part of a complicated family arrangement. Held: Pt 2F.1A - keep balance between facilitating the bringing of derivative actions and protecting Co from too ready interference in its internal management. Applicant bears onus of showing on BOP the s237(2) requirements been fulfilled 1) Co probably not taking proceedings: if unclear, onus is on applicant to prove that actual refusal is to be inferred 2) Good faith: (must satisfy even if applicant has no financial interest/present involvement with co) i) whether applicant honestly believes that a good cause of action exists and has reasonable prospect of success reasonable person test; ii) whether applicant is bringing the suit for a collateral purpose which would amount to abuse of process collateral purpose = also a competitor or creditor

Both factors interrelated - if applicant does not hold i) then it would lead to the conclusion that application was made for a collateral purpose so as to abuse of courts process; If applicant holds i) but not ii) b/c it was obtained for some gaining some advantage, it is abuse of the Courts process

24

If applicant is a former SH with nothing obvious to gain directly from the success in derivative action, the court will scrutinise with particular care the purpose for which action is said to be brought (e.g. maybe applicant has a private vendetta against company = no good faith)

If applicant, under the proceeding would be seeking to receive a benefit which he should not receive, the application will not be made in good faith even if co will benefit too.

Held: improbable that S would have appointed P as her own individual adviser to investigate Hs financial affairs if she had any misgivings about his independence, integrity and capacity to conduct the investigation properly S was not acting in good faith 3) Best interests of Co - high threshold, must be is in best interest, not maybe, appears to be Applicant generally needs to provide evidence in showing:

a)

Character of the company if family/small private company, important to consider effect of proposed litigation on purpose of company and family members as SH; or maybe JV when venturers would be deadlocked if proposed proceeding allowed as for purpose of vindicating one sides action rather than to achieve useful results.

b) c) d)

Evidence of the business so that effect of litigation may be appreciated Whether there are other avenues other than litigation available e.g. Talisman Technologies where P could obtain order for SP of contract rather than involve co into proceeding. Evidence as to ability of defendant to meet substantial part of the judgment in favour of Co from the action

The Shareholders Personal Action: p611


SH may not bring an action in their own interest in respect of a wrong done to the company, but if the wrong was in breach of a right that is personal, then they can sue

3 sources of rights:
1) Contractual rights of members, 2) Personal right recognised under statute and constitution 3) Rights given to affected parties to seek injunctions or damages

Statutory Contract in The Constitution S140(1) Effect of Constitution and Replaceable Rules A companys constitution and any replaceable rules that apply to the co have effect as a contract: (a) Between the company and each member; AND (b) Between the company and each director and company secretary; AND (c) Between a member and each other member Under which each person agrees to observe and perform the constitution and rules so far as they apply to each person.

Therefore, any legal action directly based on the constitution or rules will be an action to enforce a statutory contract. Privity of contract rules will apply [Eley].

Privity of contract rules will apply [Eley] A member can only enforce a provision if it affects the member in their capacity as a member, and not in any other capacity [Hickman v Kent or Romney Marsh].

25

Hickman v Kent or RMSB Assoc 1915: p613 statutory contracts in constitution Facts: Association was incorporated to encourage breeding of sheep and maintenance of breed Article 49 of the constitution provided for reference of disputes to arbitration P was a member of the association, dispute arose and he issued writ to have association restrained in various ways from acts in derogation of his rights. Association issued a summons asking that all further proceedings be stayed, on ground that art.49 constitution a submission to arbitration Held: The company could enforce the statutory agreement in art 49 against P in his capacity as a member only. P brought the suit in his capacity as a member, and thus he and the company are both parties to the statutory contract. 1. A member can only enforce a provision if it affects the member in their capacity as a member, and not in any other capacity

2. 3.

Constitution cannot be binding between Co and 3rd Parties, nor on a member in their capacity NOT as a member Articles regulating rights and obligations of members do create rights and obligations between members and between members and Co.

The Scope of Individual SH Rights (actions): p617


A member may have a personal right of action to complain about corporate misconduct:

Under an express contract between the member and other members, or between the member and company - may include implied terms Pursuant to a personal cause of action conferred by statute or general law Enforce the constitution as a statutory contract under s140

A personal action is not prevented by a statutory derivative action [s236(3)]. Whether a derivative or personal action should be utilised will depend on whether the right is personal, or belongs to the co.

Cause of action Conferred by Statute or General Law Members of company enjoy certain rights and obligation, if infringed, the existence of the contract does not have to be shown, case law has given the member a right. These include:

Right to vote at GM unless denied by the constitution [Pender v Lushington] Derivative suit for the vindication of corporate rights (Ngurli) Right to truly informative notice of meetings [s249J] Right to have voting rights protected against improper action of directors this may extend to protection against proper dilution [Residues Treatment and Trading] If a director holds a fiduciary duty to an individual shareholder. Rectification - s175 gives a company or person aggrieved a right to have a register maintained, including the register of members under s169 corrected.

26

- Grant v John Grant & Sons: rectification of register to remove reference to shares allotted by an improperly constituted board

- Ngurli Ltd v McCann: rectification order where contested issue was in breach of Ds duties

Injunctive relief usually with declaratory orders (s1324) Power of allotment, standing under s175 (rectification), SH can seek action to prevent issue of further shares where proposal not in good faith for benefit of comp or where Ds act in excess of their powers (Ashburton Oil v Alpha Minerals)

Residue Treatment and Trading Co v Southern Resources 1988: p619 Facts:

Board of SR made a takeover bid of Square Gold, acquiring 100% interest in return for an allotment of shares in Southern Resources. This gave McDougall and associates (D of SR and large SH in Square Gold) a larger shareholding in SR - ensured board control and frustrated takeover from Empire Mines in relation to SR. Plaintiffs were minority shareholders in SR.

Issue: Whether SH in a limited liability company has standing to challenge an allotment of shares made by directors for an improper purpose. Held:

An exception to Harbottle is where there is an infringement of personal rights, allotment made for improper purpose is not capable of been ratified by GM since it diminishes voting power which is personal right. Voting rights are a fundamental attribute of membership, and are rights which the member should be able to protect by legal action against improper diminution.

If there is standing, it must be because the SOC raises a cause of action based upon the infringement of the plaintiffs personal rights (which Ds with fiduciary obligations from nature of the contract between company and members).

SHs personal right is founded in equity, and is a right to have the say in the company which accrues by virtue of voting rights attached to his shares by contract to company, preserved against improper actions by co. or directors

Refer to Coleman duty owed by directors is to the company not SH, exception is when there is a specific relationship (like dependence on directors) Effect of Ratification In Residues Treatment, it affirmed the decision that an individual SH could sue personally, or in a derivative action upon an irregularity capable of being cured by ratification, such as matters concerning internal management of co. However, where the allotment infringes the personal rights of SH, the potential of ratification cannot deprive SH of standing, whilst the infringement continues. Standing will exist until infringement is expunged by ratification. Members Remedies The usual remedy for a member who complains of breach of the statutory contract is a declaration or injunction. SH will NOT be awarded damages against the company for breach of contract [Re Addlestone Linoleum Co].

27

Other remedies include an order for payment of a liquidated sum such as a dividend, declared but not paid. Court has no jurisdiction to rectify a constitution, even if their terms do not accord with what is proved to have been the intention of signatories [Scott v Frank Scott]

Chapman v E-Sports Club 2000 Collateral Purpose [SM 158] Facts: C commenced proceedings on behalf of ES against EW (director) for transfer of intellectual property to EW without agreement of Cs brother (legal representative of C). Cs brother swore an affidavit there was no affidavit from C himself. Held: C did not show enough evidence of his presence at meeting (believed he was not present), merely claimed minutes inaccurate, did not show it was in best interest of company as he said very little about the company. Court believed C is attempting to use proceeding as advice to put pressure on the other parties to buy him out (extricate him financially from this investment). Charlton v Baber 2003 NSWSC Facts: Charlton and Baber were founders and sole directors of NAA, C ceased to be director but remained in office and NAA later went into liquidation. C in his only capacity as SH now, sued B on several grounds: issuing shares, paying dividends to some class of shares only, excessive remunerations Held:

Fiduciary duty pleaded: the fiduciary duties having identical content cannot be owed both to the company and to one or more of its SH in relation to the same subject matter; to the extent that they exist at all, fiduciary duty owed by D to SH can be recognised only where they would not compete with any duty owed to the company.

The fiduciary duties must be pleaded specifically and in such a way as to be both circumscribed and defined by allegations of fact giving rise to some form of relationship involving ascendancy or influences and vulnerability or dependence requiring subordination of personal interest [Charlton did not indicate in his statement of claim any factual basis for this claim, so no personal damages].

Claim for derivative proceedings: C claimed breach by Baber of fiduciary duty owed to NAA in respect of allegedly excessive wages and benefits made available by NAA to Baber himself.

S237(2)(a) probability of the companys bringing the proceeding: NAA is currently in the hands of liquidators, the administrators Babers remuneration formed part of a wage splitting arrangement which split Mr and Mrs Bs wages for tax purposes, therefore it is not excessive probably that NAA under their direction as liquidators would not itself bring the proceeding S237(2)(b) Whether Charlton is acting in good faith NAA would need to satisfy secured creditor, then Mr and Mrs Baber have priority. Charlton is the only shareholder and will receive nothing in winding up unless it produces a return exceeding total shortfalls in creditors therefore nothing here shows hes not acting in good faith. C may have motives that go beyond mere personal gain, as a former D, he may feel a sense of responsibility to creditors who have suffered loss. S237(2)(c) The best interest of the company Best interest is an expression concerned with a persons separate and independent welfare.

28

NAA is insolvent, so the best interest in these circs reflects entirely or at least predominantly interests of general body of creditors Charlton wishes to have the company institute present genuine prospects of a result that will enhance returns to creditors and promote the welfare of creditors.

Some of the claims may involve relatively small returns (e.g. NAA made uncommercial loans to another company), thats not to conclude the pursuit of the claim is not in the best interest of the company provided that in the overall context (especially relates to costs), some potential benefit can be foreseen

S237(2)(d) Serious question to be tried (merits type decision) Loan on uncommercial terms and without any or adequate security, but there is nothing to show what the terms actually are, so the court cannot conclude in any satisfactory way that D breached duties.

In relation to uncommercial loan: no serious Q. the interest rate = 15% is to be distinguished from uncommercial such as no interest and no security [Q: whether loans are repayable on demand or at some fixed future time will have a bearing on the general question of whether there is a discrepancy in value b/w what the lender gave and what the lender received in return]

Excessive remuneration: not really, problem with pleading is that it did not contain anything to show
why the remuneration and benefits of Mrs and Mr B might be said to be excessive, in the sense that corporate resources were paid to D and wife without any countervailing benefit to the company by way of services or in return for services that could not be regarded as commanding such remuneration and benefits.

Improper payment of dividends to B and holder of B class shares: yes, according to constitution

17. Shareholders remedies: Oppression

Re Tivoli Freeholds Ltd 1972 VSC: p642 - Elements of SH oppression: 1. 2. Person claiming it must establish that it was unfair, in a way that was burdensome, harsh and wrongful to other member of the company and lacked degree of probity theyre entitled to expect. Oppression may occur even if all the members are treated equally, e.g. unfairness where it was by reason of advantage to parent company

29

3.

Oppression result from some overbearing act/attitude on part of oppressor, course of conduct designed to advance their own interest

To determine whether conduct is unfair it is necessary to examine it from the POV of both alleged oppressed and oppressor who acted oppressively must be shown to have acted at lease unfairly

Statutory remedy:
Under Pt 2F.1, court may grant a much wider variety of remedial orders under s233 to members where the affairs of the company are being conducted in a manner oppressive to them.

S232 Grounds for Court Order for Oppressive Conduct The Court may make an order under s232 if: (a) The conduct of a company's affairs; or (b) An actual or proposed act or omission by or on behalf of a company; or (c) A resolution, or a proposed resolution, of members or a class of members of a company; is either: 4 elements

(d)

Contrary to the interests of the members as a whole; or

(e) Oppressive to, unfairly prejudicial to, or unfairly discriminatory against, a member or members
whether in that capacity or in any other capacity.

S234 Who Can Apply for an Order? An application for an order under s233 in relation to a company may be made by: (a) A member of the company, even if the application relates to an act or omission that is against: (i) The member in a capacity other than as a member; OR (ii) Another member in their capacity as a member; OR (b) A person who has been removed from the register of members because of a selective reduction; OR (c) A person who has ceased to be a member of the company if the application relates to the circumstances in which they ceased to be a member; OR (d) A person to whom a share in the company has been transmitted by will or by operation of law; OR (e) A person whom ASIC thinks appropriate having regard to investigations it is conducting or has conducted into: (i) The companys affairs; OR (ii) Matters connected with the company's affairs.

The terms oppressive, unfairly prejudicial or unfairly discriminatory overlap and should be read compositely [Thomas v HW Thomas Ltd]. TEST Whether objectively in the eyes of a commercial bystander, there has been unfairness, namely conduct that is so unfair that reasonable D who consider the matter would not have thought the decision fair [Morgan v 45 Flers Ave Pty Ltd].

30

Whether a reasonable D weighing the furthering of the corporate object against the disadvantage, disability or burden, which their decision will imposes, would consider the decision not to be fair to impose it? [Wayde]

Whether a decision is fair, cannot be assessed in a vacuum or simply from the perspective of one member it will depend on weighing and balancing the conflicting interests of different groups within the company [Thomas v HW Thomas]. Prima facie, it is for the board to decide whose interests should prevail [Wayde].

i) Restriction on Dividend Payments Re G Jeffrey (Mens Store): p648


2 companies bequeathed to 2 sons R and A (30%), 2 daughters (10%) and widow (20%). Each son became MD of a company but R took over both companies because company languished under A. R bought shares of his sister and treated the company as his own A excluded from management. Held: Oppression need not be of members in their capacity as SH, if oppressed is a member, it is sufficient that the oppression be of him in any capacity other than that of a member. Unfairly prejudicial to and unfairly discriminatory against (acting in discriminatory manner is to act in manner which makes a distinction/difference b/w one SH and another) broader than oppressive so less onerous for applicant SH to prove the conduct he/she alleged. Construing the retention of profit, as long as MD was acting bona fide and acting in a manner in which he honestly believed was in the best interest of the cos members judge unprepared to take action As no active wrong has been done to petitioner the business is conducted in substantially same manner prior to inheritance and applicant was not discriminated again, treated in same manner as other members. There is nothing unfair in requirement to abide by the decisions of the majority of directors or shareholders. Refusal of other shareholders to purchase As shares (i.e. locked-in) did not amount to conduct of affairs of the company in a manner prejudicial or discriminatory. A tried to persuade R to buy out his shares but R offered prices too low, A sought to wind up co.

ii) Locked In Thomas v H W Thomas Ltd p650


Founder was MD which made article to confer all powers to him. A (grandson) now MD holding 40 shares. M held 1000 shares, and was not an employee of the company. Other members of the family held balance of shares. Financial management of company was conservative with modest dividends, M unsuccessfully moved a motion for certain assets to be sold, and invested in income-earning investments. He then wished to sell shares, but nothing further ensued. Held: Fairness was determined by looking at history and structure of company, and reasonable expectations of members, weighing conflicting interests of different groups within the company thus to have regard to principle governing the duties of D in conduct of affairs of co and the rights and duties of majority SH in relation to minority.

31

Background of company is relevant (family company) to explain why those in control genuinely wished to carry same business as in past, M is entitled to contend that applying ordinary commercial criteria that proposed course is unfairly detrimental to his interests.

It is not unreasonable for the company to conclude to continue trading as in the past. Assumed company carried on for financial benefit, but does not mean pursuing revenue profits is overriding goal as family company, such considerations will have to be borne in mind. Therefore, because of property ownership, potential returns, and that other members are satisfied no oppressive conduct illustrated as under s233.

Coombes v Dynasty Pty Ltd: SM178


Facts: C was a minority SH of Co, majority of shares were held in interests associated with T. Co claimed that T had caused affairs of co to be run in manner oppressive and unfairly prejudicial to C. These included a) unfair nature of proposed restructuring of co, b) failure to keep C informed of affairs and meetings and c) using money owed by C to company as a means of exerting leverage over C to purchase his shares for amount less than full value Held: The restructuring of the company done in manner which disregarded minority SH rights, incl. watering down the interests of the minority SH and the use of company assets as security for a project associated with the majority SH was oppressive and unfairly prejudicial. Other signs of oppression: failure to provide financial information, notify AGM, failure to disclose information about recent valuation, use debt owed by minority to persuade them to sell their shares [Scottish Co-op]. Only relief which was likely to give adequate protection and remedies were orders of compulsory purchase this should not be refused because there is a mechanism for disposal of shares provided in the articles. C sought compulsory purchase of shares pursuant to s260

iii) Discriminatory and Prejudicial Decision Wayde 1985 HC: p644 examine the scope of remedial orders made when oppressive conduct exist
NSWRL incorporated to conduct rugby league competition, objectives incl. fostering and control of the game and such action as may be conducive to its best interests. Authorised its board to determine which teams should participate in competition, decided to reduce number of teams from 13 to 12, and Wests application was rejected. Wests brought action seeking relief against decision Held: The board was expressly constituted to promote best interest of the sport. There was no suggestion that the board failed to have regard to the relevant considerations, decisions were made in exercise of power that is conferred on board. The board possessed special expertise and experience and power was exercised bona fide and properly not unfairly prejudicial action. The decision to reduce the number of competitors and exclude Wests was taken with full knowledge of disability, but this was weighed against the dangers of a larger competition. Mere prejudice/discrimination against member insufficient, not unfair for D in good faith to advance objects of Co to prejudice a member.

32

That reasonable D weigh the furthering of the corporate object against the disadvantage, disability or burden which their decision will impose, and address their minds to the Q whether a proposed decision is unfair.

The test of unfairness is objective court must determine whether a reasonable D possessing any special skill, knowledge or acumen possessed by the directors and having in mind the importance of furthering the corporate object on the one hand and the disadvantage, disability or burden which their decision will impose on a member on the other, would have decided that it was unfair to make that decision.

A valid resolution (adopted in good faith and for a purpose within power) is relevant but not conclusive of Q whether relief should be granted under s233. Range of Orders Available: p654

S233 Orders the Court Can Make

(1)
an order: (a) (b) (c)

The Court can make any order it considers appropriate in relation to the company, incl That the company be wound up; That the company's existing constitution be modified or repealed; Regulating the conduct of the company's affairs in the future; For the purchase of any shares by any member or person to whom a share

(d) (e)
share capital; (f) proceedings; (g)

in the company has been transmitted by will or by operation of law; For the purchase of shares with an appropriate reduction of the company's For the company to institute, prosecute, defend or discontinue specified Authorising a member, or a person to whom a share in the company has

been transmitted by will or by operation of law, to institute, prosecute, defend or discontinue specified proceedings in the name and on behalf of the company; Court may direct company to institute specific proceedings or authorise member to do so on behalf (h) (i) specified act; (j) Requiring a person to do a specified act. Appointing a receiver or a receiver and manager of any or all of the Restraining a person from engaging in specified conduct or from doing a

company's property;

The courts obligation is to grant whatever relief is necessary best suited to deal with that conduct. There is no limit on the grant by the court of an appropriate remedy. However, the court should avoid assumption of responsibility for the management of the company [Jenkins].

1. Order That Company Be Wound Up


In general, a company will not wind up a company, as it would be unfairly prejudicial to SH [Re Spargos Miing NL].

33

2. Order Regulating Conduct of Affairs of the Co In Future


In Re Spargos, court not only removed the incumbent D, but appointed certain persons as Ds. It is not necessary for court to be reluctant to interfere where necessary or desirable to give effective relief [Jenkins v Enterprise Gold Mines NL]

Re Spargos Mining NL: p655


Companys property and resources had been transferred to related companies which were almost entirely devoid of any discernible commercial benefit to it and were of benefit only to other members of the group, so relief under Pt 2F.1 established, therefore attention turned to form of relief. Held: orders made included the appointment of independent board of the courts choosing, charged with the investigation of past defaults and bringing appropriate recovery proceedings. In general, court will not interfere with ordinary day-to-day management of company but can appoint receiver (s233(1)(h)) whilst dissension in company is sorted out (but this would make company default on mortgages, can alter articles of company s233(3)). There will be no winding up where company remains viable and satisfactory company with chance of recovering provided temporary space to allow proper investigation, replace existing board with a board of judges choosing for 12 months. Appointed directors to provide independent management to avoid appointing a receiver. This was to allow investigation of past malfeasances.

Jenkins v Enterprise Gold Mines NL 1992 WASC: p657


Once the relevant oppression/unfairness is found, it is obligation for the court to grant whatever relief is best suited to deal with that conduct with no limits to the courts power. Court has the power to make orders regulating the conduct of the affairs of the company in the future, which involves the making orders which may interfere with the internal administration of the company. Pt 2F.1 did not suggest that court should be reluctant to interfere where that is necessary or desirable to give effective relief, there is express power to order appointment of receiver/manager.

3. Order for Purchase of Shares s233(1)(d) and (e)


Where an applicant desires to leave the company, the most usual order is one for the purchase of the applicants shares. Valuation of share price must be fair on the facts of the particular case [Coombs]. It will depend on the time of valuation, which will depend on circumstances. The purchase price may be: Fixed by reference to the value they would have had at date of petition if there had been no oppression [Scottish Co-op v Meyer] Partnership was valued on the basis that it was a wholly owned subsidiary [Bright Pine Mills] Where directors had taken excessive remuneration, shares were valued by capitalising dividend stream from a typical year after adjustment to restore emoluments to commercially justifiable levels [Sandford v Sandford Courier Services]. Order for relief from oppression by means of capital reduction should not be made if there are significant creditors, OR at least not be made without notifying them. [Coombes]

34

18. Winding Up
2 statutory remedies for minority shareholders:
i) Oppression of injustice ii) Compulsory liquidation remedies

External administration of insolvent companies:

Winding up appointment of liquidator (de-register company, power of D cease company focus on the interests of the creditors almost exclusively, chase up debtors and determine whether directors have conformed with their role in administration of company)

Voluntary administration (initiated by company directors and they appoint administrator, if the company is insolvent or likely to become insolvent in some future time) Pt 5.3A Receivership - appointment of someone to manage the business, power restricted to receive property and income and not to undertake management task such as asset disposal Creditors scheme of arrangement reconstructure of debts (not limited to insolvency or loan default)

Compulsory Liquidation Remedies: p129


To wind up a company, members may: Voluntarily: By passing a special resolution s490 declaration of solvency Petition a court to compulsorily wind up the company under s461.

However, as a wider variety of remedies are available under the statutory oppression remedy under s232, remedies will typically be sought upon this ground. S462(2) Standing To Apply For Winding Up Subject to this section, any one or more of the following may apply for an order to wind up a company: a) b) c) d) the company; or a creditor (including a contingent or prospective creditor) of the company; or a contributory; or the liquidator of the company; or ASIC pursuant to s464; or ASIC (in the circumstances set out in subsection (2A)); or APRA. (2A) ASIC may apply for an order to wind up a company under s462(2)(f) only if: a) b) the company has no members; and ASIC has given the company at least 1 month's written notice of its intention to apply for the order. (3) A person being, or persons including, APRA may only apply for an order to wind up a company if: a) an inspector has been appointed to make an investigation in respect of the company under s52 Insurance Act 1973 ; and

e) f)
g)

35

b)

The company's liabilities, worked out for the purpose of the prudential standards (within the meaning of that Act), exceed the company's assets worked out for that purpose.

(4) The Court must not hear an application by a person being, or persons including, a contingent or prospective creditor of a company for an order to wind up the company unless and until: a) such security for costs has been given as the Court thinks reasonable; and A prima facie case for winding up the company has been established to the Court's satisfaction.

b)

(5) Except as permitted by this section, a person is not entitled to apply for an order to wind up a company. S461(1) - Grounds on which company may be wound up by Court if: a) The company has by special resolution resolved that it be wound up by the Court; or c) Co does not commence business within 1 yr from its incorporation or suspends its business for a yr; d) The company has no members; or e) Ds have acted in affairs of the company in their own interests rather than in the interests of the members as a whole, or in any other manner whatsoever that appears to be unfair or unjust to other members; or f) affairs of the company are being conducted in a manner that is oppressive or unfairly prejudicial to, or unfairly discriminatory against, a member or members or in a manner that is contrary to the interests of the members as a whole; or g) an act or omission, or a proposed act or omission, by or on behalf of the company, or a resolution, or a proposed resolution, of a class of members of the company, was or would be oppressive or unfairly prejudicial to, or unfairly discriminatory against, a member or members or was or would be contrary to the interests of the members as a whole; or h) ASIC has stated in a report prepared under Division 1 of Part 3 of the ASIC Act that, in its opinion: i. ii. the company cannot pay its debts and should be wound up; or it is in the interests of the public, of the members, or of the creditors, that the company should be wound up; j) if the application was made by APRA--the Court is of opinion that it is in the interests of the public, of the members or of the creditors that the company should be wound up; or k) The Court is of opinion that it is just and equitable that the company be wound up. (2) A company must lodge a copy of a special resolution referred to in paragraph (1)(a) with ASIC within 14 days after the resolution is passed.

Compulsory Liquidation Remedies: p625


i) The Just and Equitable Ground (s461k)

In Ebrahimi, HL stressed that this provision confers upon the court a discretionary power of a

very side character, and that courts should be ready to apply it to new situations. Equity will subject the exercise of legal rights to equitable considerations. That is, it will restrain

the holder of common law rights from insisting on legal rights where considerations of a personal character arises between individuals, would make it unjust to do so.

Re Tivoli Freeholds Ltd 1972 VSC: p627

36

T Ltd managed theatres which had been damaged by fire so T sold and them and ceased theatrical activities. Company funds were then employed by principal SH in corporate raiding activities. Minority SH opposed current direction of company policy. Held:

As the companys main objects were concerned with entertainment, and funds had been applied

for a wholly foreign purpose to objectives and intention of members, it was just and equitable for the company to be wound up. Ratio: Test for just and equitable Question of fact and have to look at the circumstances. If the company engages in acts which are entirely outside what can fairly be regarded as having

been within the general intention and common understanding of the members when they became members. Before substratum (permanent subject of qualities) found to be withdrawn, business in practical sense = impossible.

Refer to general intention and common understanding among ALL members of the company

(found in the Constitution which states companys objects), also question of corporate capacity Companys course of conduct relevant to remove ambiguity but would not prevail against the

conclusion drawn from Constitution.

Ebrahimi v Westbourne Galleries Ltd 1973: p631


Facts:

N and E formed a private company to sell carpets. This was later incorporated with both

becoming directors. Ns son later joined the company, becoming a director and SH, good profits were made, distributed as remuneration rather than dividends.

Held: -

N and son passed a resolution removing E as a director, E wants to wind up on just and

equitable grounds Winding up the company was just and equitable in the circumstances: Ns refusal to acknowledge E as a director effectively repudiated the relationship E lost his right to a share in the profits through remuneration, and was at the mercy of N to receive dividends.

A company, however small or domestic, is a company not a partnership or even quasi-partnership and it is through the just and equitable clause that obligation, common to partnership, may come in. The court drew an analogy between small closely held companies and partnerships. This recognized that it is the essence of the relationship that mutual confidence be maintained. If this has ended, so that they can no longer work together in the way originally contemplated, the relationship should be ended.

If P can prove, some special underlying obligation of his fellow member(s) in good faith, or confidence, that so long as the business continues he shall be entitled to management participation, an obligation so basic that, if broken, the conclusion must be that the association must be dissolved.

However, a loss in trust and confidence is insufficient in itself to justify dissolution on a just and equitable ground, and requires more [International Hospitality Concepts]. It is irrelevant if the power is exercised in the bona fide interests of the company.

37

Power to exercise legal rights could be restricted on equitable grounds: Equitable considerations will operate where [Ebrahimi]:

Association formed or continued on basis of personal relationship involving mutual confidence, e.g. where partnership is converted to limited company Agreement or understanding that some (for there may be sleeping members) or all SH shall participate in conduct of business Restriction upon transfer of members interest in the company, so that if confidence is lost, the stake cannot be liquidated.

Applications of Quasi-Partnership Rule in Australia: p634


Note: cases indicate the scope of the clause for subjecting legal rights to equitable considerations Re Caratti Holdings
The constitution gave power to governing D to acquire shares of other members at par value. Although exercise of power was formally valid, to permit D to acquire the shares compulsorily at gross undervalue was contrary to a clear understanding that existed from the outset that SH would have 10% partnership interest in the business.

Kokotovich v Wallington
The minority SH was granted small shareholding at formation, holdings was recognition of pre-existing intimate and business relationship. Due to the continued animosity between the parties, and risk of oppression and limited nature of companys activities, winding up was granted.

Re City Meat Co
SHs were members of a family who acquired their shares through inheritance. Trading generated few profits and rarely paid dividends i.e. cant sell, vote or receive dividend so lost all rights as SH. The majority shareholder and MD received income but ignored the rights, expectations and obligations of other SH, justifying winding up.

Re Dalkeith Investments
Shares were divided between former spouses and daughter, and it was held the family company was in substance a partnership in a corporate form. Because of divorce, there was a loss in mutual trust and confidence and it was no longer possible for members to work together for the common good or to rely, for the protection of interests and investment in the company. Therefore, winding up permitted similarly, serious and operative state of mistrust and disharmony between the 2 D (McMillan).

Directors acting in their own interests: p637


S461(e) objective standard, whether the directors have acted in their own interests or otherwise unfairly or unjustly. This ground is complemented by s461(f) and (g) and their terms parallel those in which the oppression remedy in Pt 2F.1 is now cast.

Re Cumberland Holding Ltd 1976- statutory interpretation

When it refers to directors does not mean the board unanimously, it will be met where it is

shown that the effective majority has acted in its own interests or in the interests of one or more of

38

those board members, or even where 1 D has caused his will to be carried into effect with the effect that his personal interest has been preferred.

The affairs of the company wide operation, not limited to business or trade matters but

encompass capital structure, dividend policy, voting rights, consideration of takeover offers, and indeed, all matters which may come before the board for consideration.

Own interests even if D acted in the interest of another company of which they are also

directors and SH. The interests of the members as a whole shows directors preferred their own interests to

the interests of one or more or some significant section of the members. 1st interpretation: not interest common to ALL members but interests of one or more of the members are sufficient NOT preferred interpretation 2 nd interpretation : i) D preferred their own interests to the interest of one or more or perhaps some significant section of the members. ii) The conduct does not have to be shown to be unfair or unjust to the members as a whole, sufficient if it is unfair/unjust to at least to any significant body of other members or perhaps to any other member. Re Weedmans Ltd 1974: p638

Facts: S secured voting control and became majority of W, directors then issued substantial

block of shares to Ss subsidiary in abuse of their fiduciary power. They facilitate a takeover bid by S for outstanding equity of W (bid price grossly less than true price), they recommended that minority SH of W accept the offer (along misinformed and misled them). Judge held the recommendation for acceptance of gross undervalued big did not itself involve breach of Ds duty.

The directors failed to observe the requisite standard of commercial morality, the effect of their

failure, considered against the whole background, reacted unfairly and unjustly against other members. If allotment of shares stood in isolation, petitioners can use other remedies (set aside allotment) The nature of the injustice or unfairness, and the extent, to which this operates to the detriment but no relief available under Pt2F.1 then winding up granted. of any other member or members, will no doubt be material for consideration by the court in exercising its discretion whether or not to make a winding up order.

39

19. Corporate Regulation: The role of ASIC


S1311 - General penalty provisions (1) A person who:

a) b)
c)

does an act or thing that the person is forbidden to do by or under a provision of this Act; or does not do an act or thing that the person is required or directed to do by or under a provision of this Act; or otherwise contravenes a provision of this Act;

Is guilty of an offence by virtue of this subsection, unless that or another provision of this Act provides that the person: d) e) is guilty of an offence; or is not guilty of an offence.

(1A) Para(1)(a)-(c) only apply to a provision in the following list if penalty is set out in Sch 3: (a) Chapters 2A, 2B and 2C; (b) Parts 2F.2 and 2F.3; (c) Chapters 2G, 2H, 2J, 2M (other than Part 2M.4), 2N, 2P and 5A; (d) Parts 5B.1 and 5B.3; (da) Chapter 6CA; (db) Chapter 7; (dc) Chapter 8; (e) Chapter 10. Civil Penalty Provisions Pt 9.4 B - What Are The Civil Penalty Provisions? S180(1) S181(1), (2) S182(1), (2) S183(1), (2) S209(2) S588G(2) Duty to act with reasonable care and diligence Duty to act in good faith in the best interests and for a proper purpose

Related parties Insolvent Trading

Civil Penalty Procedure


ASIC can seek a pecuniary penalty order, disqualification order or a compensation order. Co can ONLY seek a compensation order.

i) Declaration of Contravention
Court must make a declaration of contravention as outlined in s1317E(2) [s1317E(1)]. Only ASIC or a company damaged by contravention of a civil penalty provision may seek these orders [s1317J] E.g. s1317(1)(ja) - continuous disclosure [applies to s674(2), 675(2),(2A)]; ss180, 181 officers duty; s209(2) related party; s588G(2) insolvent trading; Proceedings must be started within 6 years [s1317K]

40

Pecuniary Penalty Order

Under s1317G, after a declaration of contravention has been made, ASIC may apply to a court to order of payment of up to $200 000 to the Cth government if contravention: Materially prejudices the interests of the company or its creditors; Materially prejudices the companys ability to pay its creditors ; OR Is serious

Compensation Order
Under s1317H, company or ASIC (s1317J) may seek compensation for damage resulting from a contravention of a civil penalty provision. The test for contravention and damages is causations but for and other tortious tests s1317H(2) one must choose between account for profit OR damages.

Disqualification Order
Under s206C, ASIC may apply to a court who may disqualify a person from managing companies for a period considered appropriate, if satisfied that this would be justified. The court may have regard to:

Persons conduct in relation to the management, business or property of any other company; AND Any other matters that the court considers appropriate Criminal Penalty Provisions

Directors Duties [s181, 182, 183] Related Parties Transactions [s208] Insolvent Trading [s588G]

Under s184(1), where a director or officer commits an offence recklessly, or intentionally dishonest, they may be liable criminally. Where involvement in the contravention of s208 is dishonest, this constitutes an offence. [s209(3)] An offence will be committed where a director of a company dishonestly fails to prevent a company from incurring a debt, and suspects company was insolvent at that time [s588G(3)]

Breach can lead to fine of up to $200 000 or imprisonment of up to 5 years or both [Sch 3] S184(1) Good Faith, Use of Position and Information Criminal Offences A director or other officer of a corporation commits an offence if they:

a)
b)

Are reckless; OR Are intentionally dishonest;

AND fail to exercise their powers and discharge their duties:

c)
d)

In good faith in the best interests of the corporation; OR For a proper purpose

S1318 Power to Grant Relief If in any civil proceedings against a person for negligence, default, breach of trust or duty AND it appears that the person may be liable BUT has acted honestly, having regards to the circumstances of the case, the court may relieve the person either wholly or partly from liability on such terms as the court thinks fit.

Applies to general law duties Honestly appears to be used as involving the absence of moral turpitude

41

Unlikely to be used in favour of a director who benefits from breach of duty Will apply to s588G which imposes duty to prevent insolvent trading

Gilligan G Civil Penalties and the Enforcement of Directors Duties 1922 [SM 180- 190]
Regulators engagement with the enforcement pyramid; Strategic Regulation Theory pyramid model

Role: advocates regulatory compliance as best secured by persuasion rather than legal enforcement, stimulates maximum levels of regulatory compliance (start by assuming regulated are willing to comply voluntarily.

Sanctions: enforcement and preventative

Roles of ASIC in enforcement of Ds duties (pyramid, from bottom to top)

(i)
(ii) (iii) (iv) (v) (vi) (vii) (viii)

Persuasion and education; Negotiation and settlement Investigations, inspections and examination protective and deterrent function Letters of warning and penalty notice Remedial civil law based remedies Pecuniary Criminal and Civil penalties Management Banning Orders apex of the corporation laws enforcement pyramid Incapacitation Civil and Criminal Penalties -

Bottomley S, The Birds, the Beasts and the Bat [SM 191] - justification for corporate regulation has to be in relation to the role of the corporate form in contemporary society: Private Conception of a Company (contract-based theory):
The company is built on liberalism, free choice, utilitarian theory; the company should be left alone by the state as this will achieve efficiency; company as a nexus of contracts. Social Conception of a Company (concession theory): Suggests that the creation of a body corporate and associated legal incidents is a concession granted by the state. If this is considered to be a privilege, it underlies the states claim to control the process of incorporation and its subsequent use A weaker form is that regulation is limited to quid pro quo for the grant of corporate status.

Criticisms:
Does not fit with realities of modern corporate life administrative registration process Capital markets which discipline contracts do not require more legal intervention; no justification for subjecting companies to higher level of regulation than other contract-based theories (K).

Choice between concession and K theory provides limited choice and stereotyped positions, leaving regulation to either market or state untenable Constitutional Theory: Amalgamation of both concession and K perspectives, recognising there is a role for state regulation of corporations whilst acknowledging companies are institutions in which individual choices are formed, and collective decisions made. It considers the corporation as a constitutional arrangement. Recognises that companies are social enterprises and polities in their own right. The means by which companies are and how they govern should be constituted by state and corporate inputs. State should protect potential threats to liberty, recognising that companies are part of the wider community and thus public values should be protected.

42

Corporate relationships should be structured by their own values state should not be sole source of corporate governance rules.

Fisse and Braithwaite, Corporations, Crime and Accountability 1993 [SM 198]
The 2 major problems arising out of the attempts to control corporate wrongdoing committed by larger scale organisations:

Non-prosecution of individuals: An undermining of individual accountability at the level at which public enforceability occurs since prosecutors (etc) tend to go for corporate liability rather than individual liability since it is somewhat easier.

Non-assurance of internal accountability within organisations: When corporations are caught for wrongdoing, they are, in theory supposed to react by using internal disciplinary systems but this rarely occur, that is the corporate form can be used to obscure and deflect responsibility.

Why accountability is important: Individual accountability is essential to social control but is today more of an exception rather than a rule in respect of offences committed on behalf of larger scale organisations. Mill said responsibility is null when nobody knows who is responsibleto maintain it [responsibility] at its highest, there must be one person who receives the whole praise of what is well done, the whole blame of what is ill. The ability to fix blame is important for maintaining a credible society however the multilayered bureaucracies of large corporations enable blame to be diffused amongst a number of individuals so that it is in effect levied on none at all. If the corporate form is used to obscure and deflect responsibility, whether intentionally or unintentionally, the growth of corporate activities in industrialised societies poses an acute risk of escalating breakdown of social control. This breakdown is already patent in domains like tax compliance and toxic waste regulation. Solutions: The legal system should recognise corporate systems of justice and fully utilise their power it is only within corporations that responsibility can be identified and sheeted home.

Corporations have the capacity, but not the will to deliver clearly defined accountability for law breaking. Courts of law have the will but usually not the capacity. Thus the firms justice system must be harnessed to determine who was responsible for the breach of the law and the justice system should make them accountable.

The law should hold an axe over the head of a corporation that has committed the actus reus of a criminal offence. This may be almost literally an axe that ultimately can deliver the sanction of corporate capital punishment liquidation, deregistration of the firm charter to operate. The private justice system of the firm must then operate under the shadow of that axe.

Australian Securities and Investment Commission p57

Independent regulatory agency legislative implementation and routine administration of the Corporations Act (take civil enforcement actions and initiate criminal prosecutions and exercise some adjudicative power)

Acts as a market regulator, its overriding concern with protection of the integrity of financial markets often favours prompt intervention through civil enforcement of injunctive or administrative relief. Accountable only to the Cth Minister and Parliament, Cth Ministers may give ASIC a written direction about policies and priorities but only after giving ASICs chairperson an adequate opportunity to discuss the need for the proposed action.

43

DISCERNING THE CORPORATE MIND

Primarily: DIRECTING MIND AND WILL Where a person is the embodiment of co and their actions are really the actions of the co itself. Who is the directing mind and will? Those who speak and act as the co, typically in the higher echelons of management, and control what the co does NOT those who are its servants, or the hands of the co, and receive orders from above Directing minds may delegate functions, so that the delegate can then embody the mind and will of co. [Tesco Supermarkets] Where the directing mind acts against co interests Where the directing mind completely ceases to act in the interests of the co., it will no longer be the directing mind of the co. [Canadian Dredge] Knowledge of corporation A corporation can possess knowledge through: The knowledge of the directing mind and will can be imputed to the corporation [Houghton v Nothard] Under the law of principal and agent, co can be imputed with its agents knowledge where they are authorised to receive such information

VICARIOUS Liability Under CL, no criminal liability can be ascribed to the master for the acts of a servant unless the act was authorised, or he aided and abetted. However, there may be vicarious liability for statutory offences, which impose strict liability upon the co for the acts of servants, whether these are authorized or not. The co will be liable if the act is in fact committed by the agent with the requisite mens rea [Mousell Bros]. Must examine the contents of the STATUTE in regards to the: Object of the statute Nature of the duty imposed Person upon whom duty is imposed The person who would ordinarily perform duty Who the penalty is imposed upon Vicarious liability will arise even if the act done in co name is fraudulent [Meridien Global Funds] Who has PRIMARY liability actor or corporation? Corporate liability may be attributed to corporate officers BOTH as a principal in addition to the cos liability OR as an accessory [s79]. Whether the co AND/OR actor is primarily liable or merely an accessory will depend on the provisions of the relevant statute: Where the law speaks to the actor AND contemplates the co to be vicariously liable for the act of the actor BOTH co and actor will be primarily liable [Mallan v Lee] Where law addresses itself to co directly, co is contemplated to be the principal offender so actor is liable only for aiding and abetting [Hamilton v Whitehead] Where the law speaks directly to the actor, holding it primarily liable, and the liability of the co will be that of aider and abetter [Lewis v Crafter]

Mousell Bros v London & Northwestern Rly Co (202) Act required owners of goods for tolls, failure would incur a penalty. Co misdescribed goods consigned with intent to avoid payment of toll; co claimed that it was not criminally liable for acts of servants HELD:

Interpretation of the Act is that owner is to be responsible, fault of servant makes the owner liable

44

Corporation, being the principal, no mens rea necessary to make it liable, so it is in exactly the same position as a principal who is not a corporation

Lennards Carrying Co v Asiatic petroleum - p204 Ships cargo was lost at sea due to vessels unseaworthy condition. L was MD of Co (ships owner) and knew of ships sea unworthiness but did not take steps to prevent it going to sea. HELD:

The statutory defence without his (Co here) actual fault or privity applies i.e. fault or privity of L (officer/director) who is not merely a servant or agent for whom the company is liable upon the footing respondent, but somebody for whom the company is liable b/c his action is the very action of the company itself.

Tesco Supermarkets v Nattrass - p206

s11 (2) Act - any person offering to supply Goods and Services misdescription - criminal liability; s24 (1) defences: (a) due to default of another person (b) he took all reasonable steps T were selling powder, marked to be sold at 2s11d, C saw ad and went to buy it, but had to pay 3s11d. R, shop assistant was meant to put the specially marked packs in stock; she did not take out the higher priced ones which she should have told manager. Manager was to be responsible for getting the right stock but didnt check either

HELD: Directing mind and will: directors and managers of Co, this can be delegated to officers/employees 1) Defence of a) another person:

It was the failure of C (manager) to inspect which caused the breach, directors and managers represent the directing mind and will of Co (Tesco). Directors may delegate some part of their function and delegate can act as the Co, board here never delegated any part of functions - they only set up a chain of command through to supervisors, the manager here was not as a delegate so not directing mind and will, so C was another person

2) defence b) he took all reasonable steps:

He refers to the board, Co has taken all due diligence to ensure that this manager should run effectively, and then failure on part of manager ought not to be attributed to Co.

Meridian Global Funds v Securities Commission - p210

K was chief investment manager of M; he wanted to gain control of ENC and misused his authority and Ms funds to acquire shares in ENC in Ms name. Act requires disclosure of substantial security holder as soon as person knows or ought to know, K did not report it to M and hence M did not disclose to Commission - commission prosecuted M.

HELD:

Exception to Tesco: if Act embodies a different identification to attribute liability Not every act on behalf of Co could be expected to be subject of a resolution of board; Q of whether it is attributable to Co is question of construction of the Act. For the purposes of deciding whether Co was in contempt, the act, state of mind of employee who entered into arrangement in the course of his employment should be attributed to Co

45

Act here: where person is authorised by Co to acquire shares, such obligation is implied on the Co, otherwise, policy of Act would be defeated because Co can then allow employees to acquire interests on their behalf and they can pay as little attention as possible to what the managers were doing

Canadian Dredge and Dock v Queen (213)

Co was charged with conspiracy to defraud from bid rigging of dredging contracts; bid-rigging was performed by the officer in charge of its dredging operations. The officers were president, VP and general manager of Co

HELD:

Principal of attribution of criminal actions to Corp only operates where the directing mind is acting within the scope of his authority, where the criminal act is totally in fraud and resulting in a benefit exclusively to the employee-manager, the employee directing mind ceases to be a directing mind of the corporation and acts could not be attributed to Corp

Exception to Tesco: if act was done for personal fraud - Directing mind is only attributable to Co if:

a)
b) c)

was within the field of operation assigned to him (i.e. delegated) was not totally in fraud of the corporation was by design or result partly for the benefit of the Co

Enforceable undertakings = s93AA ASIC Act, where Co comes to ASIC and list out the things they will do - if Co fails to comply, that is in itself an offence. Corporate Criminal Liability

The problems experienced by CL in trying to hold corporations liable for criminal offences is that a corporation in incapable of any act of understanding, and it has no will to exercise an abstraction according Lennards case. It was incapable of carrying out any act except through its agents. The problems inherent in a strict approach of this nature have been obviated in tort and contract by means of the broad doctrines of authority and vicarious liability, thus a company is responsible for torts committed by its agents during the course of their employment (note issues with regards to contractors working for a company). 1. Vicarious Criminal Liability for the defaults of agents acting within the scope of their employment Civil liability and certain criminal offences. There are 3 exceptions to common law bar on vicarious criminal liability: Common law offence of public nuisance Common law offence of criminal libel Certain statutory offences which impose strict liability upon the company whether the acts were authorised by the company or not.

Moussel Bros case. The company is absolutely liable for the acts committed by its servant when the offence is statutory and the statute imposes strict liability on the company for the acts of its servants. Atkin LJ outlines the determinants of whether an Act imposes strict liability on the company in respect of an offence:

The object of the statute, words used to describe the offence The nature of the duty laid down

46

The person upon whom it is imposed, person who would ordinarily perform the duty The person on whom the penalty is imposed.

The servant does not need to be the directing mind and will of the company as in primary liability and the corporation doesnt need mens rea.

The development of vicarious criminal liability is now severely restrained by the general dislike courts have for reading a statutory criminal offence as one not requiring a mens rea such approach having its origins in Woolmington v DPP which predated the Moussell Bros case. 2. Primary liability of the company itself for criminal offences and torts In Australia there has been little clear authority on the proper bases of corporate criminal liability. The organic theory

In Lennard, Viscount Haldane LC recognised that a company may be liable for acts done by persons who can be said are the directing mind and will of the corporation, the very ego and centre

of the personality of the corporation. They are not agents, but an organ of the co and their
acts/states of mind are not their own, rather they are the acts and states of mind of the company.

This person may be the board of directors, a person under the direction of GM or they may be a person who has a co-ordinate authority with the board who is elected and retained at the hest of the general meeting.

Corporate mens rea Courts have developed a concept of corporate mens rea through reliance on primary liability doctrine.

Tesco: a company will be criminally liable for the guilty mind of a person who is the directing mind and will of the company. What this means:

The state of mind of managers of the company has to be distinguished from those of the servants of the company. Thus the co will only be liable for the acts of those who might be construed as representing the directing mind and will of the company.

This will normally be the board, the MD or other senior executive officers of the company and those to whom they delegate their discretionary powers (as long as that delegation is unreserved and the persons can act independently of the devolver).

In this case the manager of the store was not a delegate of the board or a manager. The board never delegated here, it merely set up a chain of command through regional and district supervisors. The board remained in control. The shop managers have to obey the general directions of the board so the acts or omissions of the shops managers are not the acts or omissions of the company itself.

Canadian Dredge: a person ceases to be the directing mind and will of the company when they cease completely to act (in fact or in substance) in the interests of the corporation. It seems that a company can be convicted of manslaughter: R v ICR Haulage [1944] KB Generally the companys criminal liability will be in addition to that of its human agent (which is probably a really bad word to use to describe that person given the emphasis of courts in this area on the point that the person who is the directing mind and will of the company (such as to impose the liability on the co) is not an agent.

47

Note also the personal liability of corporation officers under legislation for things like occupational health and safety (OH&S Act 1983 (NSW) s 15). Where a company breaches the Act each of its board members is also deemed to have breached it as well unless they can satisfy the Court that He or she was not in a position to influence the corporations conduct He has used all due diligence to prevent the contravention

20. Review by an examination of corporate groups


In general, a corporate group refers to a number of companies which are associated by common or interlocking shareholdings, allied to unified control or capacity to control [Walker v Winbourne]. There are few legal restrictions on the ability of business to conduct affairs through corporate groups and determine their own group structure. The degree of financial and decision-making autonomy of various group companies can vary, with power centre of corporation possibly located in the ultimate holding company, or further down chain. Legal Conception of the Corporate Group S50 Related Bodies Corporate Where a body corporate is: a) A holding company of another body corporate OR b) A subsidiary of another body corporate; OR c) A subsidiary of a holding company of another body corporate; The first mentioned body and the other body are related to each other

Under ss46-50, there may be vertical, cross or diagonal relationships between companies. S46 What is a Subsidiary? A body corporate (first body) is a subsidiary of another body corporate, if and only if: (a) The other body: (i) Controls the composition of the first bodys board; OR

(ii) Is in a position to cast, or control the casting of, more than one-half of the maximum
number of votes that may be cast at a general meeting of the first body; OR

(iii)

Holds more than one-half of the issued share capital

(b) The first body is a subsidiary of a subsidiary of the other body

Whether or not a company is a subsidiary depends on the legal definition, and not based on the corporate practices [Bluebird Investments v Graf]. A company will be a subsidiary of the holding company, regardless of whether there are other subsidiaries between itself and the ultimate holding company [s49].

48

Control of Board Composition S47 Control of a Body Corporates Board Without limiting other modes of control of board composition, a company controls the composition of the board of a second company if it has the power to appoint or remove all or the majority of the directors of the 2nd company, even if consent of another person is required. The other body is taken to have power to make such an appointment if: (a) Person cannot be appointed as a director of first body without the exercise by the other body of such a power in the persons favour; OR (b) Persons appointment as a director of the first body follows necessarily from the person being a director or other officer of the other body

For the purposes of s46(a)(i), control must flow from a legally enforceable power; practical or de facto control is insufficient in the absence of such power [Mount Edon Gold Mines v Burmine Ltd].

Mount Edon Gold Mines v Burmine Ltd p180


Facts: B and E merged and both agreed B as the subsidiary, with this reflected in the financial statements. E dispatched takeover offer for B directors advised SH not to accept bid, E argued that B was a subsidiary E had 38.5% holding in B and its nominees had been in majority on the board. Held:

There was no control E had practical or de-facto control of GM and thus board due to its substantial holding in B and because the rest of the shares were held in disperse holdings. However, this was not a legally enforceable power; it did not own over 50% of the shares.

Facts alone do not create a subsidiary-holding company relationship.

Lifting the Corporate Veil to Attack Holding Companies

The corporate veil may be lifted to hold parent companies responsible for the debts of their subsidiaries, where the corporate form has been used improperly or if one company is the agent, trustee or partner of another company.

However, control or domination of a subsidiary by a parent not sufficient to justify disregarding the separate legal personality of each group company - courts will not lift the corporate veil merely because it is a corporate group.

Briggs v James Hardie p190


B contracted asbestosis, Asbestos - shareholders were Hardies and Wunderlich in equal shares. B claimed that by employing in Asbestos acted as an agent for the other defendants, or alternatively, that he was entitled to look beyond corporate veil and sue shareholders and controllers of Asbestos. Issue: Statute of Limitations extend if evidence established a cause of action majority held that evidence existed to extend the limitation period (refused to answer corporate veil Q). Held:

Court held that there was no precedent establishing cases where the corporate veil could be lifted so JH not liable for subsidiary in torts (exceptions: fraud and injustice, where action done deliberately with specific intent to escape liability for specific tort).

In dissent, Meagher JA suggested that the corporate veil may be lifted in tort cases, especially in the law of employers negligence:

49

The potential to exercise control over subsidiary insufficient to pierce veil where one company exercises complete dominion and control over another Referred to cases in US where the corporate veil had been lifted if the parent dominated the subsidiary to the point it had no separate existence, or utilised to circumvent the law i.e. more than control.

Qintex Aust Finance Ltd v Schroders 1990 NSWSC: p195


Qintex best indicates the reserved nature of the Australian courts to lift the veil. Qintex was clearly hiding behind the corporate veil: which applied, but the decision of the court founded on the principles of contract. The court overtly expresses need for law reform for conglomerate entities Rogers CJ:

The development of the rigid application of the separate legal entity principle to corporate groups is problematic, often resulting in a divergence between the realities of commercial life and the applicable law.

Piercing the corporate veil doctrine had a larger role to play in the case of corporate groups, as long as it is used appropriately and fairly distributes the burden and benefit of the corporate form amongst the various corporate actors.

The court spoke of fairness when he stated that, Creditors of failed companies encounter difficulty when they have to select from among moving targets. This means by deduction the target is the holding company.

De Facto Director: p391


S9 Definition of Director/Officer A director is a person appointed to the position of director, or alternate director. Unless a contrary intention appears, a person who is not validly appointed as a director will be a director if they: Act in the position of a director [de facto director] Directors of the company are accustomed to act in accordance with the persons instructions or wishes [shadow director] An officer is a director or secretary of the corporation, OR A person who participates in making decisions which affect the whole, or a substantial part of the business Has the capacity to significantly affect the cos financial standing Receiver/Receiver-Manager, Administrator, Liquidator, Trustee

Whether a person is a de facto director will depend upon the particular role they have played within co. It may arise in the absence of a valid appointment, or where a person describes themselves as a consultant and it is a question of fact whether a person is performing specific functions OR is engaged in the affairs of the company in manner distinctive as a director [DCT v Austin]. For a person to be a de facto director, it was held in DCT v Austin: It was a necessity but not a sufficient condition that person exercises the top level of managerial functions Whether a person has acted as a director is a question of degree to be determined, looking at the operations and circumstances of the co including:

50

Size of the company more discretion to deal is likely to be given in large companies, and not considered to be a director Internal practices and structure of the company How putative director was reasonably perceived by outsiders

In Harris v S, D had practical direction of the company, and was its driving force, the court considered D a de facto director. Shadow Directors

This is directed at relationships in which control is unconscious, contested or carefully disguised; such as where 3rd persons control or influence is so complete that directors effectively abdicate in their favour. It covers those who are real, though not nominal controllers of the company, and may apply to a holding company, a controlling shareholder or even the nominee directors of a corporate shareholder.

When Is A Person a Shadow Director?

v S]

For a person to be a shadow director, it must be shown that it was his will, and not the

independent will of the appointed directions, which determined the resolutions of the board. [Harris

I.e. where is the locus of effective decision making? If this resides in a 3rd person, and they

cannot secure Advisor protection of s9, they will be a director [AS Nominees]. It is NOT sufficient to have a substantial shareholding of mere effective control (s50AA), or have nominee directors to constitute a shadow director - requires ACTUAL CONTROL. Does not require that there be directions or instructions embracing all matters involving the board. It only requires that as and when the directors are instructed, they are accustomed to act as the section requires [AS Nominees]

Requires directors who fulfil their role and function as directors, BUT who carry out that role in accordance with directions or instructions given by someone AND that directors perform positive acts, not simply to desist from acting. It will NOT apply where control or influence is so complete that they effectively abdicate in their favour [Harris v S].

Standard Chartered Bank v Antico p392


Giant was a listed company, in which Pioneer had 42% shareholding and 3 nominee directors A, Q and G on its board. Following default by Giant, Bank sought to recover from Pioneer, as a person who had taken part in the management of the co whilst insolvent (for providing finance to G). Held: Took part in management merely owning shares is insufficient, need factors such as effective control, implementing financial reporting requirements. Pioneer had effective control, by virtue of the size of its shareholding in comparison to the other substantial shareholders. The conditions imposed upon Giant, following the decision to fund Giant, show a willingness and ability to exercise control, and an actuality of control over its management and financial affairs. Imposed financial reporting requirements consistent with requirements for its group

51

4 March instruction that no new financial commitments be made without Pioneer approval, Decision to fund Grant on basis of security was effectively made by Pioneer and accepted by

and all payments had to be approved Giant, and never the subject of careful consideration by Giant.

Australian Securities Commission v AS Nominees p397


ASN a trustee company of superannuation funds formed by Windsor, with common boards of directors. ASC commenced proceedings to wind up 3 companies and allege that co had been run largely at direction and interest of Windsor with extensive breaches of trust

Issue: Whether W is person in accordance with whose directions/instructions the directors of ASN are accustomed to act

Held:

In ascertaining where the control lies: no need to be shown formal direction was given; and question would be where the locus of effective decision making is. Found that Windsor exercised control of a strategic character, which defined the context in which the co operated. Where Windsor intruded, the boards did not exercise independent role. The case was not merely one of directors acting on advice of a manager in proper Constant and Securities loans were for the benefit of cos under his direct control Directors acted without due deliberation, recklessly entering into transactions introduced by Windsor peremptorily sacked the board of Ample, after a disagreement between Windsor and performance of duties

Windsor the Board.

Liability of a holding company for insolvent trading by a subsidiary: p445


Elements of Contravention
S588V(1) When Holding Company Is Liable A corporation contravenes this section if: (a) The corporation is the holding company of a company at the time when the company incurs a debt; AND

(b)

The company is insolvent at that time, or becomes insolvent by incurring that debt, or

by incurring at that time debts including that debt; AND (c) One or more of the following subparagraphs applies: (i) The corporation, or one or more of its directors, is or are aware at that time that there are such grounds for so suspecting: (ii) Having regard to the nature and extent of the corporations control over the companys affairs and to any other relevant circumstances, it is reasonable to expect that: (A) (B) A holding company in the cos circumstances would be so aware; OR One or more of such a holding cos directors would be so aware

Recovery of Compensation
S588W(1) Recovery of Compensation For Loss Resulting From Insolvent Trading Where: (a) A corporation has contravened s588W AND

52

(b) (c)
suffered AND (d)

The person to whom the debt is owed has suffered loss or damage in

relation to the debt because of the cos insolvency AND The debt was wholly or partly unsecured when the loss or damage was The company is being wound up

The cos liquidator may recover from the holding co, as a debt due to the company, an amount equal to the amount of the loss or damage.

Defences to Liability
S588X Defences

(2)

It is a defence if it is proved that, at the time when the debt was incurred, the

corporation and each relevant director had reasonable grounds to expect, and did expect, that the company was solvent at that time, and would remain solvent even if it incurred that debt. (3) It is a defence if it is proved that at the time when the debt was incurred, the co, and (a) Had reasonable grounds to believe and did believe: each relevant director

(i)

That a competent and reliable person was responsible for providing to the co adequate information about whether the co was solvent, AND

(ii) Person was fulfilling that responsibility AND

(b) (4)

Expected on the basis of information provided to the co by the person, that

the co was solvent at that time and would remain so, even if it incurred that debt. If it is proved that, because of illness or for some other good reason, a particular

relevant director did not take part in the management of the co at the time when the co incurred the debt, the fact that the director was aware is to be disregarded

(5)

It is a defence if it is proved that the co took all reasonable steps to prevent co from

incurring debt.

Reasonableness is assessed with regard to factors such as the size and complexity of the co concerned, the size of debt incurred, and nature of the grounds which gave rise to the suspicion of insolvency.

53

Fiduciary Loyalty within Corporate Groups: p484


Directors To Act In Reference To The Companys Interests Directors of a company within a group are obliged to act by reference to their perception of its interests, and not that of the group generally. They cannot treat the corporate group as a single unit for the purpose of internally transferring funds or assets, or providing security. The separate legal entity of each group must be respected, and the merits of the transaction must be viewed by reference to its interests alone [Walker v Wimborne].

Walker v Wimborne p485


Liquidator of Asiatic brought malfeasance proceedings against its directors for making an unsecured loan to Australian Sound, which was in financial difficulty. The 2 companies had common directors, and administered as an integrated group in fact funds were transferred around the group on basis of need with little regard to the borrowers capacity to repay. Held: Directors breached duty, it was not in Asiatics interest to make the loan as it did not receive any commercial benefit or advantage from advancing the funds it did not suffice that the payment could have been for the benefit of the corporate group collectively. If payment of money by A to B enabled B to carry on its business, this would have derivative Where Co were not group members (no interlocking shareholdings) then group argument does benefits for A as SH of B if B enabled to trade profitably or realise its assets to advantage. not apply (here: Asiatic did not have holding of Aust) so As D held liable - D having regarded to collective welfare of the group is justified. When Can Directors Consider collective welfare of the Corporate Group? S187 Directors of Wholly Owned Subsidiaries D of a corporation that is a wholly-owned subsidiary of a body corporate is taken to act in good faith in the best interests of the subsidiary if:

(a) The constitution of the subsidiary expressly authorizes the director to act in the best interests of
the holding company; AND (b) Director acts in good faith in the best interests of the holding company; AND

(c) The subsidiary is not insolvent at the time the director acts and does not become insolvent
because of the directors act.

In Charterbridge, an objective test was applied Ds decision will not be in breach if an intelligent and honest person in position of D could have reasonably believed that the transaction for the benefit of the company. Though the court in Equiticorp applied the test in Charterbridge, they suggested: If D in fact failed to consider the companys interest breach of duty If transaction could be viewed objectively as being in the companys interest no consequence

would flow from the breach

54

Equiticorp Finance v Bank of New Zealand


Bank of NZ lent $200M to U, a wholly owned subsidiary of ETL, a member of the Equiticorp group of companies. 41% of EHL, the ultimate holding company, was controlled by Hawkins who was also chairman of the group. 2 members, EFL and EFSA had funds on deposit at Bank of NZ to reduce exposure, the bank insisted that these funds be used to reduce Us debt, and was reluctantly taken by Hawkins Subsequently, EFL and EFSA went into liquidation. Loss of Bank of NZ support would have been disastrous for Equiticorp, and welfare of group was Having regard to liquidity, and holding companys guarantee of debt, those responsible thought [Dissent] Kirby J: Though Equiticorp was not insolvent, they had liquidity problems, which an Held: intimately tied with welfare of individual companies. that the steps taken would protect the group as a whole, and thus of benefit to the companies. intelligent and honest director should give consideration to. This is because of the impact that would befall creditors and shareholders. - Proposal was for a purpose distant to companys objects exposed their vulnerable liquidity, and exposed them to greater risk of insolvency - No intelligent and honest director of EFL could have considered it the best interest of company to authorise use of funds Upstream/Downstream Loans The major issue is: does the company providing the security obtain some sort of benefit? Downstream loan this is relatively easy to discern as the parent acquires a direct or derivative commercial benefit, such as a dividend payment Upstream loan this is harder to discern. English courts have upheld loans where past and continuing stability of the group company providing the security depended on the continuing survival of the recipient group company. However, there would be a breach if the group company providing security received no real commercial benefit. The interests of creditors may have to be considered, if the transaction will affect the solvency of the company. Ratification of Breach In general, breaches of fiduciary duty can be cured by SH ratification this can be affected by ownership structure of the corporation. Therefore, where D breaches their fiduciary duties by entering into intragroup dealings, they may be protected by either prospective or retrospective ratification. Ratifying intra-group dealings will NOT constitute a fraud on the minority. The Position of Nominee Directors p489 A nominee director is person appointed on the understanding that they will represent or generally advance the interest of some other person or group. Nominee has a dual role as a director to a company, AND to represent the nominators interests: - If representative nominee: act more explicitly as the guardian of appointors interests - If independent nominee: wider altitude in exercise of judgment. It is argued that nominee directors are an adjusted form of fiduciary liability, owing both a duty to the nominator and company, thereby attenuating the fiduciary duties of directors. That is, the extent and

55

degree of fiduciary duty depends not only on the particular relationship, but the particular circumstances and terms of instrument governing exercise of a fiduciarys powers [Levin v Clark]. Though nominees owe fiduciary duties to the company they are appointed to, they can advance their nominators interests provided that in doing so, they have an honest and reasonable belief that they are acting consistently with the interest of their group company [Berlei Hestia v Fernyhough].

Levin v Clark
Facts: Companys constitution provided for appointment of defendants as governing directors, with powers exercisable only in event of Ps default under a mortgage security, given over shares in company Held: Nominee directors can act in interests of both company and the particular group so acting in The constitution, coupled with the sale and mortgage arrangements constituted an attenuation the interests of the mortgagee does not necessarily cease to act in interests of the company. of fiduciary duty of directors. This was sufficient to permit them to act primarily in the interests of the mortgagee. Additionally, this will depend on the definition of the interest of company D may not be acting in interest of SH, but in the legitimate interests of the mortgagee a stakeholder of the company. Mortgage was defaulted P challenged assumption of power on grounds that they owed primary allegiance to mortgagee and not company as a whole.

56

También podría gustarte